SlideShare a Scribd company logo
1 of 77
Download to read offline
1
DEFINITE INTEGRAL
A SHORT NOTES
Arun Umrao
https://sites.google.com/view/arunumrao
DRAFT COPY - GPL LICENSING
2 Definite Integration
Contents
1 Definite Integration 3
1.1 Definite Integration . . . . . . . . . . . . . . . . . . . . . . . . 3
1.2 Properties . . . . . . . . . . . . . . . . . . . . . . . . . . . . . 6
1.2.1 In-dependent of Variable . . . . . . . . . . . . . . . . . 6
1.2.2 Flipping of Limits . . . . . . . . . . . . . . . . . . . . . 7
1.2.3 Splitting of Limits . . . . . . . . . . . . . . . . . . . . 8
1.2.4 Shifting of Variable . . . . . . . . . . . . . . . . . . . . 12
1.2.5 Shifting of Periodic Function . . . . . . . . . . . . . . . 14
1.2.6 Even & Odd Function . . . . . . . . . . . . . . . . . . 16
1.2.7 Periodic Even & Odd Function . . . . . . . . . . . . . 23
1.2.8 Splitting of Limit in n Parts . . . . . . . . . . . . . . . 25
1.2.9 Definite Integration & Limits . . . . . . . . . . . . . . 32
1.3 Average Value . . . . . . . . . . . . . . . . . . . . . . . . . . . 34
1.4 Piece-wise Functions . . . . . . . . . . . . . . . . . . . . . . . 37
1.5 Integral Identities . . . . . . . . . . . . . . . . . . . . . . . . . 47
1.6 Special Integral . . . . . . . . . . . . . . . . . . . . . . . . . . 50
1.6.1 Definite Integral of Laplace Type . . . . . . . . . . . . 50
1.6.2 Definite Integral of Fourier Type . . . . . . . . . . . . 54
1.6.3 Specific Workout . . . . . . . . . . . . . . . . . . . . . 57
1.6.4 Repetitive Integrals . . . . . . . . . . . . . . . . . . . . 60
1.6.5 Derivative of Integrals . . . . . . . . . . . . . . . . . . 62
1.6.6 Vector Integration . . . . . . . . . . . . . . . . . . . . 64
1.6.7 Principal & General Value . . . . . . . . . . . . . . . . 65
1.6.8 Integral of Graphs . . . . . . . . . . . . . . . . . . . . 66
1.6.9 Vertically symmetric Function . . . . . . . . . . . . . . 72
1.7 Exponential Trigonometric Functions . . . . . . . . . . . . . . 77
1.1. DEFINITE INTEGRATION 3
1Definite Integration
In this chapter, the integration of infinite continuous function is bounded
between two limits.
1.1 Definite Integration
When integration of a function is limited between two fixed points where
function is continuous and limit exists everywhere between these two limits,
then The integral is called definite integration. If f(x) is a continuous func-
tion in [a, b] and limit exists everywhere in [a, b] then definite integration of
the function is given by
I =
b
Z
a
f(x) dx (1.1)
Definite integral depends on lower limit and upper limit only. It is indepen-
dent of any intermediate values between [a, b]. If F(x) is integral of f(x) and
there are intermediate points j, k and l such that [a, j], [j, k], [k, l], [l, b] then
I =
j
Z
a
f(x) dx +
k
Z
j
f(x) dx +
l
Z
k
f(x) dx +
b
Z
l
f(x) dx
and
I = F(j) − F(a) + F(k) − F(j) + F(l) − F(k) + F(b) − F(l) = F(b) − F(a)
It shows that only limits a and b are useful. Rest intermediate points are
cancelled by each other. Consider a function f(x) = x that is undergoing
integration (bar summation) in [−1, 1]. Take the width dx = 0.25, and
corresponding integral steps n = 8. The bar diagram for integral of f(x) is
given below:
4 Definite Integration
x
f
b
−1.00
b
−0.75
b
−0.50
b
−0.25
b
0
b
0.25
b
0.50
b
0.75
b
1.00
The integral in summation form is given by
I =
8
X
i=1
f(xi) × dxi
After solving the integral, we have I = 1 unit. Now, from the direct method
of integral, if F is integral of function f then plot of F versus x shall be
looked like as given below:
x
F
b
−1.00
b
−0.75
b
−0.50
b
−0.25
b
0
b
0.25
b
0.50
b
0.75
b
1.00
b
F(−1) = 0.5
b
F(1) = 0.5
As we already discussed that, the definite integral depends only on the
lower limit and upper limit of the integral. So, the I shall be equal to the sum
of F(−1) and F(1). From the plot, I = 0.5 + 0.5 = 1. In case of multi-term
function, i.e.
I =
1
Z
0
(2x − 2) dx
Never consider f(x) = 2x − 2 as a line whose slope is 2 and intercepts y-axis
at −2 to avoid numerical errors. But consider it as
I =
1
Z
0
2x dx −
1
Z
0
2 dx
i.e. difference of areas covered by 2x and x-axis, and 2 and x-axis. Here,
two areas of subtracted rather than finding the area of line curve 2x − 2
1.1. DEFINITE INTEGRATION 5
and x-axis. In later case, integral may be zero or negative depending on the
values of two areas.
Solved Problem 1.1 In the following plot, integral of function f, i.e. F and
x graph is drawn. Find
1
Z
0
f(x) dx
Again explain, why this value is negative.
Solution
1
2
−1
1 2 3
−1
−2
x
F
b
b
b
b
b
3.0
0.0
-1.0
0.0
3.0
From the integral
F = I =
1
Z
0
f(x) dx
So, from definite integral
1
Z
0
f(x) dx = F(1) − F(0)
From the plot,
F(1) − F(0) = −1 − 0 = −1
Here, negative sign represents that there may be more than one terms in
function f(x) whose integral F(x) is plotted here. During the integration of
a multi-term function, result may be positive, zero or negative, depending on
the values of area covered by first term of the function f(x) and area covered
by second term of the function f(x).
6 Definite Integration
1.2 Properties
A definite integral obeys some rules/properties which reduces integrals into
easy to solve within the domain of limits.
1.2.1 In-dependent of Variable
If function f(x) is independent of its independent variable, i.e. x, then
b
Z
a
f(x) dx =
b
Z
a
f(t) dt
Proof
R
f(x) dx represents that we have to find the sum (integrate) of
product of function f(x) and dx within the limits of x from a to b. Similarly,
the relation
R
f(t) dt has same meaning except that, here independent vari-
able is t. Both statements are true and projecting similar meaning. Hence
b
Z
a
f(x) dx =
b
Z
a
f(t) dt (1.2)
Solved Problem 1.2 Transform the function in new variable s. The function
is Z
x sin x dx
Solution Substitute x = s. It gives dx = ds. It converts the integral as
Z
x sin x dx =
Z
s sin s ds
Solved Problem 1.3 Transform the function in new variable s. The function
is Z
sin x
x
dx
1.2. PROPERTIES 7
Solution Substitute x = s. It gives dx = ds. It converts the integral as
Z
sin x
x
dx =
Z
sin s
s
ds
1.2.2 Flipping of Limits
If a and b > a are lower limit and upper limit respectively for a function f(x),
where it is being integrated then function integration takes place from a to
b. If the direction of integration is change then the integration of function
becomes negative, i.e.
b
Z
a
f(x) dx = −
a
Z
b
f(t) dt
Proof Definite integration of
b
R
a
f(x) dx is
b
Z
a
f(x) dx = F(b) − F(a)
The right hand side of above relation shows that the value of integrated func-
tion F(x) at lower limit a, i.e. F(a) is subtracted in the value of integrated
function F(x) at upper limit b, i.e. F(b). Now take common in right hand
side of above relation
b
Z
a
f(x) dx = −[−F(b) + F(a)]
and it shows that from above description stated, upper limit is a and lower
limit is b. Hence
b
Z
a
f(x) dx = −
a
Z
b
f(x) dx (1.3)
8 Definite Integration
1.2.3 Splitting of Limits
Take, a and b as lower and upper limits for the integral of a function f(x).
If F(x) is integral of f(x) as
Z
f(x) dx = F(x)
and function f(x) is continuous through the region between [a, b]. We can
add a third limit point between boundary limits, a → b, such as a → c → b,
where a < c < b. In this case, integral of the function becomes
b
Z
a
f(x) dx =
c
Z
a
f(x) dx +
b
Z
c
f(x) dx (1.4)
Proof Definite integration of
b
R
a
f(x) dx is
b
Z
a
f(x) dx = F(b) − F(a)
Adding and subtracting function value, F(c), at a point c in above relation
at right side.
b
Z
a
f(x) dx = F(b) − F(a) + F(c) − F(c)
1
1 2
x
y
a b
1
1 2
x
y
a b
c
Figure 1.1: This function is integrated in the closed limit of [a, b]. It rep-
resents area of region enclosed between function and x-axis. Function is
partitioned by splitting limit [a, b] in [a, c] and [c, b]. Area of the region in
limit [a, b] is sum of area of these two regions.
1.2. PROPERTIES 9
Rearranging right hand side of above relation
b
Z
a
f(x) dx = [F(b) − F(c)] + [F(c) − F(a)]
Rewriting right side in integral form:
b
Z
a
f(x) dx =
b
Z
c
f(x) dx +
c
Z
a
f(x) dx
b
Z
a
f(x) dx =
c
Z
a
f(x) dx +
b
Z
c
f(x) dx (1.5)
Solved Problem 1.4 Find definite integral of each function as plotted in
following figure.
Solution
1
1 2
t
y
1
1 2 3
t
y
In the first figure, curve has two line segments, (i) the line segment be-
tween x = 0 to x = 1 and (ii) line segment between x = 1 to x = 2. The
equation of lines are
y1 = x; y2 = −x + 2
Here, slope of first line segment is +1 and second line segment is −1 and the
y-axis intercepts are c = 0 and c = 2. Now the definite integral of the given
curve is
I =
Z 2
0
y dx =
Z 1
0
y1 dx +
Z 2
1
y2 dx
Substituting the values, we have
I =
Z 1
0
x dx +
Z 2
1
(−x + 2) dx
10 Definite Integration
Now, solving it, we get the definite result as
I =
1
2
+
1
2
= 1
This is first result. In the second figure, the equations of line segments can
be found by using principle of equation of line passes from two coordinate
points. Thus the line segments between definite limits x = 0 to x = 1, x = 1
to x = 2 and x = 2 to x = 3 are respectively
y1 = x; y2 = 1; y3 =
−x
2
+ 2
The integral is
I =
Z 1
0
y1 dx +
Z 2
1
y2 dx +
Z 3
2
y3 dx
Substituting the values, we have
I =
Z 1
0
x dx +
Z 2
1
1 dx +
Z 3
2
(−x/2 + 2) dx
Now, solving it, we get the definite result as
I =
1
2
+ 1 +
3
4
= 2.25
This is second result.
1
1 2
t
y
1
1 2 3
t
y
These result gives the area bounded between curve and the x-axis.
Solved Problem 1.5 Find definite integral of each function as plotted in
following figure.
Solution
1.2. PROPERTIES 11
1
1 2
t
y
1
1 2 3
t
y
In the first figure, curve has two line segments, (i) the line segment be-
tween x = 0 to x = 1 and (ii) line segment between x = 1 to x = 2. The
equation of lines are
y1 = −x + 1; y2 = x − 1
Here, slope of first line segment is −1 and second line segment is +1 and the
y-axis intercepts are c = 1 and c = −1. Now the definite integral of the given
curve is
I =
Z 2
0
y dx =
Z 1
0
y1 dx +
Z 2
1
y2 dx
Substituting the values, we have
I =
Z 1
0
(−x + 1) dx +
Z 2
1
(x − 1) dx
Now, solving it, we get the definite result as
I =
1
2
+
1
2
= 1
This is first result. In the second figure, the equations of line segments can
be found by using principle of equation of line passes from two coordinate
points. Thus the line segments between definite limits x = 0 to x = 1, x = 1
to x = 2 and x = 2 to x = 3 are respectively
y1 = −0.5x + 1 y2 = 0.5x; y3 = −0.5x + 2
The integral is
I =
Z 1
0
y1 dx +
Z 2
1
y2 dx +
Z 3
2
y3 dx
Substituting the values, we have
I =
Z 1
0
(−0.5x + 1) dx +
Z 2
1
0.5x dx +
Z 3
2
(−0.5x + 2) dx
12 Definite Integration
Now, solving it, we get the definite result as
I =
3
4
+
3
4
+
3
4
=
9
4
= 2.25
This is second result.
1
1 2
t
y
1
1 2 3
t
y
These result gives the area bounded between curve and the x-axis.
1.2.4 Shifting of Variable
If f(x) is a continuous function in [a, b] and limit exists everywhere in [a, b]
then definite integration of the function is given by
I =
b
Z
a
f(x) dx
Now shift the function by sum of limits, i.e.
x = a + b − t; dx = −dt
The corresponding limits becomes, when x = a, t = b and when x = b, t = a.
So,
I =
a
Z
b
f(a + b − t) × −dt
From the property of flipping of limits, we have
I =
b
Z
a
f(a + b − t) dt
1.2. PROPERTIES 13
From the property of changing of independent variable t → x, we have
I =
b
Z
a
f(a + b − x) dx
Here, both relation represents to the same integral and have same meaning.
This property is useful in finding of integral of relation type
R =
f(x)
f(x) + f(k − x)
Solved Problem 1.6 Evaluate
2
R
1
f(x)
f(x)+f(3−x)
dx.
Solution Let
I =
2
Z
1
f(x)
f(x) + f(3 − x)
dx (1.6)
Put x = 1 + 2 − t, it gives dx = −dt. When x = 1, t = 2 and when x = 2,
t = 1. Now, the integral relation becomes
I =
1
Z
2
f(3 − t)
f(3 − t) + f(t)
× −dt
Or
I =
2
Z
1
f(3 − t)
f(3 − t) + f(t)
dt
Transform t → x, we have
I =
2
Z
1
f(3 − x)
f(3 − x) + f(x)
dt (1.7)
Adding equations 1.6 and 1.7, we have
2I =
2
Z
1
f(3 − x) + f(x)
f(3 − x) + f(x)
dx
14 Definite Integration
Or
2I = 1 ⇒ I =
1
2
This is desired result. Note that the answer of these types of relations is
always half of the difference of upper and lower limits.
1 2 3
x
y
f(x) = sin x
sin x+sin(3.14−x)
1
−1
x
y
f(x) = cos x
cos x+cos(−x)
The function f(x) should neither change its sign (from positive y-axis to
negative y-axis or vice-versa) or nature (from sine to cosine or vice-versa)
within the limits of integration. For example, this rule is applicable for
f(x) =
sin x
sin x + sin(3.14 − x)
but it is not applicable for
f(x) =
sin x
sin x + sin(4 − x)
as sin(4 − x) is negative till 4 − x > π.
1.2.5 Shifting of Periodic Function
If f(x) is a periodic function in a range from 0 to a then the function f(x)
can be written as a
Z
0
f(x) dx =
a
Z
0
f(a − x) dx
Proof sin(x) is a periodic function of 2π. After each interval of 2π, its
phase is same to the phase, that was just before 2π value. Take a periodic
function f(x) of period a. So,
x = a − t; dx = −dt
The corresponding limits becomes, t = a when x = 0 and t = 0 when x = a.
Put the value of x, dx and corresponding limits in left side of relation
1.2. PROPERTIES 15
x
y
x
f(x)
x
y
x
f(x − a)
a
Figure 1.2: f(x) is periodic function of period a, hence function height at x
and at x ± a are same. Function f(x) and f(x ± a) have same meaning.
a
Z
0
f(x) dx =
0
Z
a
f(a − t) × (−dt)
= −
0
Z
a
f(a − t) dt
=
a
Z
0
f(a − t) dt
Changing the variable t into x
a
Z
0
f(x) dx =
a
Z
0
f(a − x) dx (1.8)
Split of Limit in Periodic Functions
Take a sine function as shown below. This function is being integrate within
limits of θ = 0 to θ = 2π.
x
y
x
f(x)
b
0 b
π
x
y
x
−f(x)
b
π b
2π
Sign of periodic function changes from positive to negative and vice-versa
continuously. For the period of θ = 0 to θ = π, sine function is in positive
16 Definite Integration
region. While for the period of θ = π to θ = 2π, sine function is in negative
region. Therefore to integrate the sine function in full range of θ = 0 to
θ = 2π is zero. It shown that area covered between horizontal line and
function is zero, which is impossible from the above figure. So, to get the
actual result, we split the limits according to the nature of the function. So,
2π
Z
0
f(θ) dθ =
π
Z
0
|f(θ)| dθ +
2π
Z
π
| − f(θ)| dθ
1.2.6 Even & Odd Function
x
f(x)
−a
a
sin x
x
f(x)
−a
a
cos x
Take the cases of sine and cosine as shown in above figure. f(x) = sin x is an
odd function as we approach to negative side of x-axis, function approaches to
negative y-axis within limits of −a ≤ x ≤ a. Therefore, it is not symmetrical
about y-axis. Again, f(x) = cos x is an even function as we approach to
negative side of x-axis, function function remains in positive y-axis within
limits of −a ≤ x ≤ a. Therefore, it is symmetrical about y-axis, hence it is
even function. A function is said to be even or odd function, if
f(x) =
(
Even if f(−x) = f(x)
Odd if f(−x) = −f(x)
If a function does not met with any of the above two conditions, then function
is neither an odd function nor an even function. Now, take a function f(x),
which is either an even function or an odd function within symmetrical limits
from −a to +a. f(x) can be transform as
a
Z
−a
f(x) dx =





2
a
R
0
f(x) dx if f(x) is even function
0 if f(x) is odd function
(1.9)
1.2. PROPERTIES 17
Proof From left side of above definition
a
Z
−a
f(x) dx =
0
Z
−a
f(x) dx +
a
Z
0
f(x) dx (1.10)
Now put x = −t in the first part of right hand side of the above relation then
x = −t; dx = −dt
The corresponding limits are obtained when x = −a then t = a. And when
x = 0 then t = 0. The corresponding result is
0
Z
−a
f(x) dx =
0
Z
a
f(−t)(−dt) (1.11)
= −
0
Z
a
f(−t) dt (1.12)
=
a
Z
0
f(−t) dt (1.13)
Case I : Even Function If f(x) is even function then f(−t) = f(t) the
relation (1.10) becomes
a
Z
−a
f(x) dx =
a
Z
0
f(t)(dt) +
a
Z
0
f(x) dx
And replacing t by x result is
a
Z
−a
f(x) dx =
a
Z
0
f(x)(dx) +
a
Z
0
f(x) dx
x
f(x)
I1 I2
−a
a
cos x
18 Definite Integration
From above figure, I = I1 + I2 where I represents to integral equivalent
to area covered within the function and x-axis. Now, I1 = I2, therefore,
I = 2I1 = 2I2. Therefore
b
Z
a
f(x) dx = 2
a
Z
0
f(x) dx (1.14)
Case II : Odd Function If f(x) is odd function then f(−t) = −f(t) the
relation (1.10) becomes
a
Z
−a
f(x) dx = −
a
Z
0
f(t)(dt) +
a
Z
0
f(x) dx
And replacing t by x result is
a
Z
−a
f(x) dx = −
a
Z
0
f(x)(dx) +
a
Z
0
f(x) dx
x
f(x)
I1
I2
−a
a
sin x
From above figure, I = −I1 + I2 where I represents to integral equivalent
to area covered within the function and x-axis. Algebraic sum of these two
areas is zero. Therefore I = 0, i.e.
b
Z
a
f(x) dx = 0 (1.15)
1.2. PROPERTIES 19
Integration of Even & Odd Functions
Simply, take a function f(x) = xn
which is defined as
f(x) =
(
Even if n is even
Odd if n is odd
On, its integration, we get
I =
Z
f(x) dx =
Z
xn
dx =
xn+1
n + 1
Now,
f(x) =
(
Even if n is odd
Odd if n is even
It means, an odd function converts into an even function and vice-versa.
Again, take the case of sin x, its integration is cos x. Here, sin x is an odd
function which is transform into even function after integration.
x
f(x)
sin x
R
x
f(x)
cos x
Solved Problem 1.7 Find whether f(x) = x2
is an even function or an odd
function.
Solution
x
f(x)
20 Definite Integration
A function f(x) is even if f(−x) = f(x) and odd if f(−x) = −f(x). Now,
substituting the x by −x in the given function:
f(−x) = (−x)2
= x2
= f(x)
Therefore, the given function is an even function.
Solved Problem 1.8 Find whether f(x) = −x2
is an even function or an odd
function.
Solution
x
f(x)
A function f(x) is even if f(−x) = f(x) and odd if f(−x) = −f(x). Now,
substituting the x by −x in the given function:
f(−x) = −(−x)2
= −x2
= f(x)
Therefore, the given function is an even function.
Solved Problem 1.9 Find whether f(t) = t3
− 2t − 4 is an even function or
an odd function.
Solution A function f(t) is even if f(−t) = f(t) and odd if f(−t) =
−f(t). Now, substituting the t by −t in the given function:
f(−t) = (−t)3
− 2(−t) − 4 = −t3
+ 2t − 4 = −(t3
− 2t + 4) 6= f(t) 6= −f(t)
Therefore, the given function is neither an odd function nor an even function.
Solved Problem 1.10 Find whether f(t) = sin t + t cos t is an odd function
or an even function..
Solution A function f(t) is even if f(−t) = f(t) and odd if f(−t) =
−f(t). Now, substituting the t by −t in the given function:
f(−t) = sin(−t) + (−t) cos(−t) = − sin t − t cos t = −(sin t + t cos t) = −f(t)
Therefore, the given function is an odd function.
1.2. PROPERTIES 21
Solved Problem 1.11 Show that integration of even function f(x) = x2
− 2
is results into odd function.
Solution The given function f(x) = x2
− 2 is an even function. Now,
integrating it, we have
F(x) =
Z
f(x) dx =
Z
(x2
− 2) dx =
x3
3
− 2x
Now, the function is F(x) = x3
3
− 2x. To check the function is odd or even,
substitute x by −x.
F(x) =
(−x)3
3
− 2(−x) = −
x3
3
+ 2x = −

x3
3
− 2x

= −F(x)
Therefore, F(x) is an odd function.
Solved Problem 1.12 Show that derivative of an odd function is an even
function.
Solution A function is said to be odd function if f(−x) = −f(x). Take
simple odd function as
f(x) = axn
⇒ f(−x) = −axn
Where n is a positive odd integer, i.e. 1, 3, . . .. Using derivative rules, we
have
f
′
(x) = an xn−1
As n is a positive odd number, hence a × n will be positive. We know that
if n is an odd number then n ± 1 is always even number. Now, n − 1 is
definitely an even number. Now,
f
′
(−x) = an(−x)n−1
= an xn−1
Hence, the derivative of an odd function is an even function.
Solved Problem 1.13 Show that derivative of an even function is an odd
function.
Solution A function is said to be even function if f(−x) = f(x). Take
simple even function as
f(x) = axn
⇒ f(−x) = axn
22 Definite Integration
Where n is a positive odd integer, i.e. 0, 2, 4, . . .. Using derivative rules, we
have
f
′
(x) = an xn−1
As n is a positive even number, hence a × n will be positive. We know that
if n is an even number then n ± 1 is always odd number. Now, n − 1 is
definitely an odd number. Now,
f
′
(−x) = an(−x)n−1
= −an xn−1
Hence, the derivative of an even function is an odd function.
Solved Problem 1.14 Show that integration of function f(x) = sin x is an
even function.
Solution The given function f(x) = sin x is an odd function. Now,
integrating it, we have
F(x) =
Z
f(x) dx =
Z
sin x dx = cos x
Now, the function is F(x) = cos x. To check the function is odd or even,
substitute x by −x.
F(x) = cos(−x) = cos x = F(x)
Therefore, F(x) is an even function.
Solved Problem 1.15 Show that definite integral of odd function f(x) = sin x
is zero in the domain of −π/2 ≤ x ≤ π/2.
Solution The given function f(x) = sin x is an odd function. Now,
integrating it, we have
F(x) =
Z
f(x) dx =
Z
sin x dx = cos x
Now the definite integral is
F(x)|
π/2
−π/2 = cos x|
π/2
−π/2 = 0
Hence definite integral of odd function f(x) = sin x is zero in the given
domain.
1.2. PROPERTIES 23
Solved Problem 1.16 Show that definite integral of even function f(x) =
cos x in the domain of −π/2 ≤ x ≤ π/2 is twice to the definite integral of
the function within the domain of −π/2 ≤ x ≤ 0 or 0 ≤ x ≤ π/2.
Solution The definite integral of the given function in the domain of
−π/2 ≤ x ≤ π/2 is
I =
π/2
Z
−π/2
cos x dx = sin x|
π/2
−π/2 = 2
Now, the definite integral of the same function in the domain of −π/2 ≤ x ≤
0 or 0 ≤ x ≤ π/2 is
I1 =
0
Z
−π/2
cos x dx = sin x|0
−π/2 = 1
and
I2 =
π/2
Z
0
cos x dx = sin x|π/2
0 = 1
This shows that I is twice to the either I1 or I2.
1.2.7 Periodic Even  Odd Function
If f(x) is either an even periodic function or an odd periodic function of
range from 0 to 2a then
2a
Z
0
f(x) dx =





2
a
R
0
f(x) dx if f(2a − x) = f(x)
0 if f(2a − x) = −f(x)
(1.16)
Proof From left side of above definition
2a
Z
0
f(x) dx =
a
Z
0
f(x) dx +
2a
Z
a
f(x) dx (1.17)
24 Definite Integration
Now put x = (2a − t) and dx = −dt in the second term of the right hand
side of the above relation alongwith corresponding limits are; when x = a,
t = a and when x = 2a, t = 0. Now, the corresponding relation becomes
2a
Z
0
f(x) dx =
0
Z
a
f(2a − t) (−dt) (1.18)
= −
0
Z
a
f(2a − t) dt (1.19)
=
a
Z
0
f(2a − t) dt (1.20)
I If f(2a − x) is even periodic function then f(2a − t) = f(t) the relation
(1.17) becomes
2a
Z
0
f(x) dx =
a
Z
0
f(t) dt +
a
Z
0
f(x) dx
And replacing t by x result is
2a
Z
0
f(x) dx =
a
Z
0
f(x) dx +
a
Z
0
f(x) dx
2a
Z
0
f(x) dx = 2
a
Z
0
f(x) dx (1.21)
II If f(2a − x) is odd function then f(2a − t) = −f(t) the relation (1.17)
becomes
2a
Z
0
f(x) dx = −
a
Z
0
f(t) dt +
a
Z
0
f(x) dx
And replacing t by x result is
2a
Z
0
f(x) dx = −
a
Z
0
f(x) dx +
a
Z
0
f(x) dx
1.2. PROPERTIES 25
2a
Z
0
f(x) dx = 0 (1.22)
1.2.8 Splitting of Limit in n Parts
If f(x) is a periodic function of period a then
na
Z
0
f(x) dx = n
a
Z
0
f(x) dx
Proof The relation becomes
na
Z
0
f(x) dx =
a
Z
0
f(x) dx +
2a
Z
a
f(x) dx + . . . . . . +
na
Z
(n−1)a
f(x) dx (1.23)
Applying properties of the periodic function, when f(x) is even function,
then the relation becomes
na
Z
0
f(x) dx =
a
Z
0
f(x) dx +
a
Z
0
f(x) dx + . . . . . . +
a
Z
0
f(x) dx (n times)
na
Z
0
f(x) dx = n
a
Z
0
f(x) dx (1.24)
It proved.
Solved Problem 1.17 Solve
π/2
R
0
√
sin x
√
sin x+
√
cos x
dx = π
4
.
Solution The integral is
π/2
R
0
√
sin x
√
sin x+
√
cos x
dx which is a periodic function in
the interval of θ = 0 to θ = π
2
. Now
I =
π/2
Z
0
√
sin x
√
sin x +
√
cos x
dx (1.25)
26 Definite Integration
And its periodic derivation is
I =
π/2
Z
0
p
sin(π/2 − x)
p
sin(π/2 − x) +
p
cos(π/2 − x)
dx
I =
π/2
Z
0
√
cos x
√
cos x +
√
sin x
dx (1.26)
Now adding above two equations
2I =
π/2
Z
0
√
cos x +
√
sin x
√
cos x +
√
sin x
dx
=
π/2
Z
0
1dx
Taking integration and substituting the limits of integration
2I = [x]π/2
0 = π/2
Now solving it we will get the result as
I =
π
4
(1.27)
Solved Problem 1.18 Solve
π/2
R
0
sin2 x
sin x+cos x
dx = 1
√
2
loge(
√
2 + 1).
Solution
Solved Problem 1.19 Solve
π
R
0
x tan x
sec x+tan x
dx = π(π
2
− 1).
Solution The integral is
π
R
0
x tan x
sec x+tan x
dx which is a periodic function in the
interval of θ = 0 to θ = π. Now
I =
π
Z
0
x tan x
sec x + tan x
dx (1.28)
1.2. PROPERTIES 27
And its periodic derivation is
I =
π
Z
0
(π − x) tan(π − x)
sec(π − x) + tan(π − x)
dx
I =
π
Z
0
(π − x) (− tan x)
− sec x − tan x
dx (1.29)
Now adding above two equations
2I =
π
Z
0
π tan x
sec x + tan x
dx
Rationalizing right hand side by multiplying and diving it by sec x − tan x.
2I =
π
Z
0
π tan x(sec x − tan x)
sec2 x − tan2
x
dx
=
π
Z
0
π tan x(sec x − tan x)
1
dx
Opening the bracket to make two parts in right hand side.
2I =
π
Z
0
π tan x · sec x dx −
π
Z
0
tan2
x dx
=
π
Z
0
π tan x · sec x dx −
π
Z
0
(sec2
x − 1) dx
Taking integration and substituting the limits of integration
2I = π [sec x]π
0 − π [tan x − x]π
0
= π [sec π − sec 0] − π [tan π − π − tan 0 + 0]
= π [−1 − 1 − 0 + π + 0 + 0]
= π [π − 2]
Now solving it, we will get the result as
I = π
π
2
− 1

(1.30)
28 Definite Integration
Solved Problem 1.20 Evaluate
π
R
0
ecos θ
· cos(sin θ) dθ.
Solution The given integral is
I =
π
Z
0
ecos θ
· cos(sin θ) dθ
It is definite integral whose limits ranges from θ = 0 to θ = 2π. Now changing
the term ecos θ
· cos(sin θ) in exponential form.
ecos θ
· cos(sin θ) = Real part of {ecos θ
· ei sin θ
}
Here cos(sin θ) is taken from relation
ei sin θ
= cos(sin θ) + i sin(sin θ)
ei sin θ
has both real and imaginary parts but according to question only real
part is required in our problem. Now
π
Z
0
ecos θ
· cos(sin θ) dθ = Real part of



π
Z
0
ecos θ
· ei sin θ
dθ



= Real part of



π
Z
0
ecos θ+i sin θ
dθ



= Real part of



π
Z
0
eexp(iθ)
dθ



Substitute eiθ
= z and eiθ
i dθ = dz
π
Z
0
ecos θ
· cos(sin θ) dθ = Real part of



θ=π
Z
θ=0
1
i
ez
z
dz



= Real part of



θ=π
Z
θ=0
1
i
1
z

1 + z +
z2
2!
+ . . . . . .

dz



= Real part of
(
1
i

log z + z +
z2
2 × 2!
+ . . . . . .
θ=π
θ=0
)
1.2. PROPERTIES 29
Substituting the value of z
π
Z
0
ecos θ
· cos(sin θ) dθ = Real part of

1
i

iθ + eiθ
z +
e2iθ
2 × 2!
+ . . . . . .
π
0

= Real part of

θ +
eiθ
i
+
e2iθ
2 × 2! × i
+ . . . . . .
π
0

Taking only real terms in right hand side
π
Z
0
ecos θ
· cos(sin θ) dθ =

θ + sin θ +
sin 2θ
2 × 2!
+ . . . . . .
π
0
= π
It is the required answer.
Addition of Constant Term in Integral Result
In indefinite integration, after each integration we add a constant value
for proper result. But the question is why do we do so. The answer is
that, in indefinite integration we take reference point “zero” for a variable.
Hence range of integration lies between zero to variable itself. If we want
to change the reference point other than zero, we need two values. (i) new
reference point, ‘c’ and (ii) function value, f(c) at this new reference point.
The function value at this new reference point, ‘c’, can be found by just
putting the new reference point value in function f(x). This constant value
is work as new reference function. In definite integration the reference point
other than zero is represented by lower limit and reference function value
is represented by the lower limit integral value. Mathematically
I =
Z
f(x) dx
is infinite integral. Its lower limit is zero and upper limit is variable of
integral itself. Hence in definite integral form, indefinite integral can be
written as
I =
x
Z
0
f(x) dx
If we want to change the reference point other than zero, we have to add
a constant term in right hand side. Let the reference point is c in x-line
30 Definite Integration
where 0  c  x then above integral becomes
I =
c
Z
0
f(x) dx +
x
Z
c
f(x) dx
here
c
R
0
f(x) dx is definite integral for x-limits ranging from zero to constant
‘c’. If it is a constant value then
I =
x
Z
c
f(x) dx + C
There is another reason that can interpret it. Suppose a function is like
f(x) = g(x) + C
After differentiation it gives
F(x) = G(x)
Where F(x) = d
dx
f(x) and G(x) = d
dx
g(x). If we again integrate above
differentiated result then we get the actual function f(x)
f(x) =
Z
F(x) dx =
Z
G(x) dx
But after integration, the result will be
f(x) = g(x)
It differs from the actual function as there is no constant term. To make it
actual function value, we add a constant term in right hand side. So
f(x) = g(x) + C
This is reason for out question. In case of definite integration, lower limit
is act as reference point, hence there is no need to add a constant term in
1.2. PROPERTIES 31
right hand side of the answer. For example
I =
x
Z
c
f(x) dx
Solved Problem 1.21 If f(x) = x6
− x3
+ 15, then find
1
R
0
f
′′
(x) dx.
Solution The second derivative of the given function is
f
′′
(x) = 30x4
− 6x
Now,
I =
1
Z
0

30x4
− 6x

dx =

6x5
− 3x2
1
0
This gives I = 3.
Solved Problem 1.22 If f(x) = tx3
− t2
x2
+ 5, then find
1
R
0
f
′′
(x) dt.
Solution The second derivative of the given function is
f
′′
(x) = 6tx − 2t2
Now,
I =
1
Z
0

6tx − 2t2

dt =

3xt2
−
2
3
t3
1
0
This gives
I = 3x −
2
3
This is desired answer.
32 Definite Integration
Solved Problem 1.23 A resonance term f(t) = sin(ωt) is derivated twice
with respect to t and then integrated with respect to ω. Find the result.
Solution The given function is
f(t) = sin(ωt)
Its twice derivative about t is given by
f
′′
(t) = −ω2
sin(ωt)
Now, integral is
I =
Z
−ω2
sin(ωt) dω
On integration it with respect to ω, we have
I = −
2tω sin(ωt) + (2 − t2
w2
) cos(ωt)
t3
This is desired result.
1.2.9 Definite Integration  Limits
A definite integration of a function f(x) withing bounded region [a, b] is given
by
I =
b
Z
a
f(x) dx
Here, a is lower bound of the function and b is upper bound of the function.
If function f(x) is integrated within symmetric boundaries as
I =
a
Z
−a
f(x) dx
then it can be represented at with limits as
I = lim
ε→a
ε
Z
−ε
f(x) dx
1.2. PROPERTIES 33
This notation is helpful, when function is defined at a point but integration is
being done in a range of limits. Integration of function at points 0 is written
as
I = lim
ε→0
ε
Z
−ε
f(x) dx
and at point ∞ is written as
I = lim
ε→0
1/ε
Z
−1/ε
f(x) dx
Solved Problem 1.24 Evaluate limε→0
ε
R
−ε
e−x
dx.
Solution This integral is symmetric integral near about zero. The inte-
grand will represent a line of zero width and certain height. So, even if the
height of the function is one unit at ε = 0, yet the area is zero due to zero
width. Now,
I = lim
ε→0
ε
Z
−ε
e−x
dx = lim
ε→0
h
−e−x
ε
−ε
i
Or
I = lim
ε→0

eε
− e−ε

x
f
x
f
x
f
Figure 1.3: The symmetry of integration in above three figures is for |ε| →
0.5, |ε| → 0.25 and |ε| → 0.05.
Expanding the exponential and simplifying them, we get
I = lim
ε→0

ε +
−ε3
3!
+ . . .

= 0
34 Definite Integration
This is desired answer.
1.3 Average Value
We know that
R
f(x) dx represents the Area between function f(x) and two
points of width dx in base line. If
b
Z
a
f(x) dx
is area between f(x) and x-limits (x = a and x = b). The average value for
this limit is given by
Iavg =
b
R
a
f(x) dx
b
R
a
dx
(1.31)
Above relation is similar to the mathematical average, i.e. average area is
the ratio of total area to the difference of limit.
Solved Problem 1.25 Find the average value of constant ‘c’ for the limits
ranges from t = a to t = b.
Solution The average value of function f(t) is given by
Iavg =
b
R
a
c dt
b
R
a
dt
t
c b b
c c
b b
a b
Now
Iavg = c
[t]b
a
[t]b
a
1.3. AVERAGE VALUE 35
Solving the relation
Iavg = c
This shows that the average value of constant remains constant.
Solved Problem 1.26 Find the average value of f(t) = t2
for the t-limits
ranges from t = a to t = b.
Solution The average value of function f(t) is given by
Iavg =
b
R
a
t dt
b
R
a
dt
t
f(t)
b
b
f(a)
f(b)
b b
a b
t line
Now
Iavg =
h
t2
2
ib
a
[t]b
a
Solving the relation
Iavg =
1
2
(a + b)
This shows that the average values of a line is average of sum of two ends of
the line.
Solved Problem 1.27 Find the average value of f(x) = x2
for the x-limits
ranges from x = 2 to x = 3.
Solution From the average relation
Iavg =
b
R
a
f(x) dx
b
R
a
dx
36 Definite Integration
x
f(x)
b
b
f(a)
f(b)
b b
a b
f(x) = x2
Substituting the values and integrating for the limits
Iavg =
3
R
2
x2
dx
3
R
2
dx
Or
Iavg =
h
x3
3
i3
2
[x]3
2
Or
Iavg =
19
3
This is the required average value.
Solved Problem 1.28 Find the average value of f(x) = sin(x) within the
angle limits in radian values from x = π/3 to 2π/3.
Solution The average of function within the limits is
Iavg =
2π
3
R
π
6
sin(x)dx
2π
3
R
π
6
dx
On integration and solving it
Iavg =
[− cos(x)]
2π
3
π
6
[x]
2π
3
π
6
1.4. PIECE-WISE FUNCTIONS 37
x
f(x)
b
b
f(π/6)
f(2π/3)
b b
π/6 2π/3
f(x) = sin(x)
Iavg =
− cos(2π
3
) + cos(π
6
)
2π
3
− π
6
Or
Iavg =
0.5 + 0.866)
2.093 − 0.523
=
1.366
1.57
= 0.87
This is required result.
1.4 Piece-wise Functions
A function F(x) is said to be a piece-wise function if it follows different
functions at different conditions. See the following function F(x), that is
defined for two conditions of independent variable x, i.e. (i) when x  0 and
(ii) x ≥ 0.
F(x) =
(
f(x) if x  0
g(x) if x ≥ 0
(1.32)
The integration of the piece-wise functions is performed within the domain
of the functions. Integration of above function F(x) is given by
Z
f(x) dx =
0
Z
−∞
f(x) dx +
∞
Z
0
g(x) dx (1.33)
Solved Problem 1.29 Find the integral of the given function.
f(x) =
(
2 if − 4 ≤ x  0
x if 0 ≤ x ≤ 2
38 Definite Integration
Solution Integration of given function is
I =
2
Z
−4
f(x) dx
Replacing the function f(x) for its domain of ‘x’
I =
0
Z
−4
2 dx +
2
Z
0
x dx
On solving it
I = [2x]0
−4 +

x2
2
2
0
Or
I = 10
Solved Problem 1.30 Find the integral of the given function in −∞  x 
∞.
f(x) =
(
x − 1 if − 1 ≤ x ≤ 0
0 otherwise
Solution
1
−1
−2
1
−1
−2
x
f(x)
1
−1
−2
1
−1
−2
x
F(x)
1.4. PIECE-WISE FUNCTIONS 39
The integral of the piecewise function is
F(x) =
∞
Z
−∞
f(x) dx
Substituting the function in above integral relation for as it is defined piece-
wise.
F(x) =
−1
Z
−∞
0 dx +
0
Z
−1
(x − 1) dx +
∞
Z
0
0 dx
On solving it
F(x) =

x2
2
− x
0
−1
= 0 −

(−1)2
2
− (−1)

=
3
2
It is integral of given piecewise function in domain −∞  x  ∞.
Solved Problem 1.31 Find the integral of the given unit step function within
the domain of 0 ≤ t ≤ ∞.
u(t − 2) =
(
0 if t  2
1 if t ≥ 2
What will be integration of e−at
u(t − 2) within the same domain of ‘t’?
Solution A unit step function is that function whose value is zero if
variable is less than the transition point. And its value is ‘1’ if the variable is
equal or larger than the transition point. In the given function, u(t − 2) tells
that the transition point for the variable t is t − 2 = 0 ⇒ t = 2. Therefore,
the unit step function value is 0 for t  2 and 1 for t ≥ 2.
1. Integration of given function is
I =
∞
Z
0
u(t − 2) dt
Replacing the unit step function u(t − 2) for its domain of ‘t’
I =
2
Z
0
0 dt +
∞
Z
2
1 dt
40 Definite Integration
On solving it
I = 0 + [x]∞
2
Or
I = ∞
Ans-1.
2. Integration of e−at
u(t − 2) is
I =
∞
Z
0
e−at
u(t − 2) dt
Replacing the unit step function u(t − 2) for its domain of ‘t’
I =
2
Z
0
e−at
× 0 dt +
∞
Z
2
e−at
× 1 dt
On solving it
I = 0 +

e−at
−a
∞
2
Or
I =
e−2a
a
Ans-2.
By this way, we can obtain the Laplace Transformations of unit step
function also. Second derivative is very important in engineering.
Solved Problem 1.32 Check whether the given piece-wise function exists or
not.
f(t) =
(
2t if 0 ≤ t ≤ 1
t + 1 if 1 ≤ t ≤ 2
Also find the
∞
R
−∞
f(t) dt.
Solution Function is critical for both piece-wise parts at t = 1. Therefore,
at this point, function should be continuous for its existence. So
f(t)−
= 2 × 1 = 2
1.4. PIECE-WISE FUNCTIONS 41
and
f(t)+
= 1 + 1 = 2
Here, f(t)+
= f(t)−
, hence function exists. It is also necessary condition for
the existence of the piece-wise function.
1
2
3
1 2
t
f(t)
(1)
1
2
3
1 2
t
f(t)
(2)
1
2
3
1 2
t
f(t)
(3)
Figure 1.4: Figure (1) is plot of piece-wise function, while figure (2) is plot
of integral of the given function. Figure (3) shows the total area covered by
given piece-wise function and x-axis.
Now, integration of the given function is
I =
∞
Z
−∞
f(t) dt
The domain of the given function is not universal, therefore, the integral
limits are depends on the sub-functions of piece-wise function. So,
I =
1
Z
0
2t dt +
2
Z
1
(t + 1) dt
Or
I = [t2
]1
0 +

t2
2
+ t
2
1
On solving it, we get I = 7/2. This is desired result.
42 Definite Integration
Solved Problem 1.33 Find the integration of function as shown below:
Solution
1
1 2 3
x
y
This piecewise function is a constant function as f(x) = 1 within the
limits of integration 1 ≤ x ≤ 3. Now, its integration is
I =
3
Z
1
1 dx = x|3
1 = 3 − 1 = 2
This is result.
Solved Problem 1.34 Find the integration of function as shown below:
Solution
x
y
a a + k
This piecewise function is a constant function as f(x) = c within the
limits of integration a ≤ x ≤ a + k. Now, its integration is
I =
a+k
Z
a
c dx = c
h
x|a+k
a
i
= ck
This is result.
1.4. PIECE-WISE FUNCTIONS 43
Solved Problem 1.35 Find the integration of function as shown below:
Solution
1
−1
1 2 3 4 5
x
y
This piecewise function is a square wave function as |f(x)| = 1. f(x) = 1
within the limits of integration 1 ≤ x ≤ 2, f(x) = −1 within the limits of
integration 2 ≤ x ≤ 3, f(x) = 1 within the limits of integration 3 ≤ x ≤ 4
and so on. Assuming that the integral is area integral, then
I = 4
2
Z
1
1 dx = 4

x|2
1

= 4
This is result.
Solved Problem 1.36 Sketch the graph of function
u(t − 1) =

0 if t  1
1 if 1 ≤ t ≤ 3
and find the integral of it.
Solution The sketch of the graph of above piecewise function is as given
below:
1
1 2 3
t
u
This piecewise function is a unit step function which makes step at t = 1.
44 Definite Integration
Now, its integration is
I =
1
Z
−∞
0 dt +
3
Z
1
1 dt = t|3
1 = 3 − 1 = 2
This is result.
Solved Problem 1.37 A ramp function is given by
f(x) =



0 if x  0
mx if 0 ≤ x ≤ a
1 if a  x ≤ a + k
Find its integral in symbolic form. What shall be the integral value if (i)
a = 2, k = 1 and (ii) a = 3, k = 0.25?
Solution
x
f(x)
a a + k
b
1
α
The integral of this ramp function shall be
I =
∞
Z
−∞
f(x) dx
Substituting the piecewise function here, we have
I =
0
Z
−∞
0 dx +
a
Z
0
mx dx +
a+k
Z
a
1 dx
On integrating it, we have
I = c + m ×
x2
2
a
0
+ x|a+k
a
1.4. PIECE-WISE FUNCTIONS 45
As the function is zero for x  0, so c = 0. So,
I =
ma2
2
+ k
This is symbolic form of result of the given integral. Now
1: a = 2 and k = 1
I1 =
m × 22
2
+ 1 = 2m + 1
2: a = 3 and k = 0.25
I1 =
m × 32
2
+ 0.25 = 4.5m + 0.25
These are desire results.
Solved Problem 1.38 A piecewise function is defined as
f(x) =

1/k if a ≤ x ≤ a + k
0 otherwise
Show that area covered by this function is unity, i.e. its integral is unity
irrespective of value of k. Plot its graph.
Solution The integral of this function is
I =
∞
Z
−∞
f(x) dx
Substituting the piecewise function here, we have
I =
a
Z
−∞
0 dx +
a+k
Z
a
1
k
dx +
∞
Z
a+k
0 dx
On integrating it, we have
I = c +
1
k
× x|a+k
a + d
46 Definite Integration
Here, c and d are arbitrary constants. As the function is zero for x  a and
x  a + k, so c = 0 and d = 0. So,
I =
1
k
× k = 1
This result is independent of k, so integration of the given function is always
unity irrespective of value of k. The plot of the given function is
1/k
a a + k
Figure 1.5: Plot of function f(x) = 1/k with k = 0.75.
Solved Problem 1.39 A piecewise function is defined as
f(x) =

1/100 if k ≤ x ≤ k + 100
0 otherwise
Show that its integral is unity. Plot its graph.
Solution The integral of this function is
I =
∞
Z
−∞
f(x) dx
Substituting the piecewise function here, we have
I =
a
Z
−∞
0 dx +
k+100
Z
k
1
100
dx +
∞
Z
k+100
0 dx
On integrating it, we have
I = c +
1
100
× x|k+100
k + d
1.5. INTEGRAL IDENTITIES 47
Here, c and d are arbitrary constants. As the function is zero for x  k and
x  k + 100, so c = 0 and d = 0. So,
I =
1
100
× 100 = 1
This shows that result is unity. The plot of the given function is
1/100
k k + 100
Figure 1.6: Plot of function f(x) = 1/k with k = 100.
1.5 Integral Identities
Let f(x) is a n degree function represented by In. The integration of this
function within limits a to b is given by
In =
b
Z
a
f(x) dx
The right hand side of above relation can be written in terms of I and n only
as
In = αIj + βIk + γ
Where Ij and Ik are the integral of the same function within the same range
of limits as of In and having degrees other than the n.
Solved Problem 1.40 Find the trigonometric identity for the integral
In =
π/2
Z
0
sinn
x dx
48 Definite Integration
Solution Let the given function is
In =
π/2
Z
0
sinn
x dx
Resolute right hand side for integral of sine function and obtaining the inte-
gral similar to given in problem but have lesser degree.
In =
π/2
Z
0
sinn−2
x sin2
x dx
Converting the sin2
x into 1 − cos2
x and
In =
π/2
Z
0
sinn−2
x 1 − cos2
x

dx
Or
In =
π/2
Z
0
sinn−2
x dx −
π/2
Z
0
sinn−2
x cos2
x dx
First term in right hand side of above relation is similar to the given integral
except that it has degree of (n − 2), now
In = In−2 −
π/2
Z
0
sinn−2
x cos2
x dx
Second term in right hand side of above integration is a beta function if n−2
is positive value. Now it gives
In = In−2 −
β 3
2
, n−1
2

2
It is the required identity of the given function.
1.5. INTEGRAL IDENTITIES 49
Solved Problem 1.41 Find the trigonometric identity for the integral
In =
π/2
Z
π/4
cotn
x dx
Solution Let the given function is
In =
π/2
Z
π/4
cotn
x dx
Resolute right hand side for integral of cotangent function and obtaining the
integral similar to given in problem but have lesser degree.
π/2
Z
π/4
cotn−2
x cot2
x dx
Converting the cot2
x into csc2
x − 1 and
In =
π/2
Z
π/4
cotn−2
x csc2
x − 1

dx
Or
In =
π/2
Z
π/4
cotn−2
x csc2
x dx −
π/2
Z
π/4
cotn−2
x dx
Second term in right hand side of above relation is similar to the given integral
except that it has degree of (n − 2), now
In = −In−2 +
π/2
Z
π/4
cotn−2
x csc2
x dx
In second term, substitute cot x = t. It gives − csc2
x dx = dt. Corresponding
lower and upper limits are 1 to 0. Now it gives
In = −In−2 +
0
Z
1
tn−2
dt
50 Definite Integration
Assuming that n − 2 is positive. Hence
In = −In−2 −
1
n − 1
It is the required identity of the given function. This identity is true for all
values of n where n ≥ 2. Now rearranging the degree of integral as n → n+1.
In+1 = −In−1 −
1
n
This is required answer.
1.6 Special Integral
In mathematics, there are two important operators, (i) gamma operator and
(ii) factorial. A gamma operator is represented by symbol Γ and factorial is
represented by | or ! symbol. For example, factorial of integer 10 is written
either as 10! or as |10. Similarly, gamma operator of a number 10 is given by
Γ(10). Mathematically
|10 = 10 × 9 × . . . × 3 × 2 × 1
In a factorial expansion, last multiplier is one. Again,
Γ(10) = 9 × 8 × Γ(8)
Gamma operator and factorial has relation as
n! = Γ(n + 1)
Note that argument of factorial operator is always a positive integer while ar-
gument of gamma operator is a real number. This is basic difference between
factorial and gamma operator. In factorials, |10.25 and |−10 operations are
invalid.
1.6.1 Definite Integral of Laplace Type
Integration, of those functions which can be transformed into exponential
form can be integrated and transform in form of Laplace Transformation. If
1.6. SPECIAL INTEGRAL 51
f(t) is a positive function of t and s is a constant variable, then integral of
type
Ilapace = L[f(t)] =
∞
Z
0
f(t)e−st
dt (1.34)
is known as Laplace integral and method is called Laplace Transformation.
Transformation word is used as old variable t is transformed into new vari-
able s, i.e. the function is transform from time domain (say t domain) into
frequency domain (say s domain). Here e−st
is Laplace operator, i.e. inte-
gral kernel and f(t) is operand. Old and new variable may be denoted by
other alphabetic letters and it does not matter in our solutions. In Laplace
transform, eat
is known as shift operator. If a is positive then s is shifted
leftward, i.e. s + a. Similarly, if a is negative then s is shifted rightward, i.e.
s + a.
Solved Problem 1.42 Find the integral of e−at
with respect to t within limits
from t = 0 to t = ∞. Also find the integral value when a → ∞.
Solution From the given problem
I =
∞
Z
0
e−at
dt =
e−at
−a
∞
0
It gives
I =
1
a
Note that, here a is taken as positive value. If a is taken as negative value
then function becomes eat
which is divergent function.
t
f(t)
a = 1
t
f(t)
a = 10
t
f(t)
a = 100
Figure 1.7: Plot of the graph of function f(t) = e−at
for a = 1, a = 10 and
a = 100 respectively. The plot drops rapidly just right to t = 0 as a increases
to infinity.
52 Definite Integration
The integral value at a → ∞ is given by
Ia→∞ =
1
∞
= 0
Solved Problem 1.43 Find the integral of t e−at
with respect to t within
limits from t = 0 to t = ∞.
Solution From the given problem
I =
∞
Z
0
te−at
dt =

−
(at + 1)e−at
a2
∞
0
It gives
I =
1
a2
Note that, here a is taken as positive value. If a is taken as negative value
then function becomes eat
which is divergent function.
t
f(t)
a = 1
t
f(t)
a = 10
t
f(t)
a = 100
Figure 1.8: Plot of the graph of function f(t) = t e−at
for a = 1, a = 10 and
a = 100 respectively. The plot become more flat rapidly as a increases to
infinity.
Solved Problem 1.44 Find the integral of e−at
, te−at
and t2
e−at
about t
within limits from t = 0 to t = ∞. Using the symmetry, find the integral of
tn
e−at
.
Solution According to the question,
I1 =
∞
Z
0
e−at
dt =
e−at
−a
∞
0
=
1
a
1.6. SPECIAL INTEGRAL 53
I2 =
∞
Z
0
te−at
dt =

−
(at + 1)e−at
a2
∞
0
=
1
a2
I3 =
∞
Z
0
t2
e−at
dt =

−
(a2
t2
+ 2at + 2)e−at
a3
∞
0
=
2
a3
From these three results, we find that the integral values are in order of
In =
n!
an+1
Where, n is power of t and is an integer. So, the
∞
Z
0
tn
e−at
dt =
n!
an+1
This is desire result.
Solved Problem 1.45 Let κ is an operator which means
κ[f(x)] =
t
Z
0
f(x) dx
Now, find the value of κ sin x and κ2
sin x.
Solution The given relation is
κ[f(x)] =
t
Z
0
f(x) dx
Now, substituting the value of f(x) = sin x, we have
κ[sin x] =
t
Z
0
sin x dx = [− cos x]t
0
It gives κ[sin x] = 1 − cos t. Now, κ2
[sin x] can be written as κ [κ[sin x]].
Therefore, f(x) becomes κ[sin x] and its value is 1 − cos x, (t is replaced by
54 Definite Integration
x). Now
κ2
[sin x] =
t
Z
0
(1 − cos x) dx = [x − sin x]t
0
It gives κ2
[sin x] = t − sin t.
Solved Problem 1.46 Find the value of
R ∞
0
e−x2
dx.
Solution To find the integral of this problem, substitute x2
= t. The
corresponding 2x dx = dt. Now, when x = 0, t = 0 and when x = ∞, t = ∞.
The integral becomes
I =
Z ∞
0
t−1/2
e−t
dt
2
Or
I =
1
2
Z ∞
0
t
1
2
−1
e−t
dt =
1
2
× Γ

1
2

This gives I =
√
π/2.
1.6.2 Definite Integral of Fourier Type
We know that Laplace type integral of a function is given by
IL =
∞
Z
0
f(x)e−sx
dx
in s-domain or time domain. If s is replaced iω or jω (where i2
= −1 or
j2
= −1) then integral becomes Fourier type.
IF =
∞
Z
0
f(x)e−iωx
dx (1.35)
Where ω = 2πf in frequency domain. Laplace type integral is related with
Fourier type integral by s ←→ iω. For complete Fourier transform
IF =
∞
Z
−∞
f(x)e−iωx
dx (1.36)
1.6. SPECIAL INTEGRAL 55
A discrete function can also be represented into continuous function by apply-
ing Fourier Series method, i.e. converting Fourier type integral into discrete
series. A function f(x) is equivalent to
f(x) = a0 +
∞
X
n=1
an cos(nx) +
∞
X
n=1
bn sin(nx) (1.37)
Where a0, an and bn are three Fourier constants which can be evaluated for
definite limit range x = a to x = b by using following three relations.
a0 =
1
b − a
b
Z
a
f(x) dx (1.38)
an =
2
b − a
b
Z
a
f(x) cos(nx)dx (1.39)
bn =
2
b − a
b
Z
a
f(x) sin(nx)dx (1.40)
Solved Problem 1.47 Find the constant value of Fourier series integration of
the given function f(x) = x within limits from 0 to π.
Solution Constant term a0 in the Fourier series integration is given by
a0 =
1
b − a
b
Z
a
f(x) dx
Taking limits from 0 to π and function f(x) = x, the Fourier series constant
integral term becomes to
a0 =
1
π − 0
π
Z
0
x dx
Or
a0 =
1
π

x2
2
π
0
It gives a0 = π/2.
56 Definite Integration
Solved Problem 1.48 Find the constant value of Fourier series integration of
the given function f(x) = sin x within limits from 0 to π.
Solution Constant term a0 in the Fourier series integration is given by
a0 =
1
b − a
b
Z
a
f(x) dx
Taking limits from 0 to π and function f(x) = sin x, the Fourier series con-
stant integral term becomes to
a0 =
1
π − 0
π
Z
0
sin x dx
Or
a0 =
1
π
[− cos x]π
0
It gives a0 = 2/π.
Solved Problem 1.49 Find the constant value of Fourier series integration of
the given function f(x) = x sin x within limits from 0 to π.
Solution Constant term a0 in the Fourier series integration is given by
a0 =
1
b − a
b
Z
a
f(x) dx
Taking limits from 0 to π and function f(x) = x sin x, the Fourier series
constant integral term becomes to
a0 =
1
π − 0
π
Z
0
x sin x dx
Or
a0 =
1
π
[sin x − x cos x]π
0
It gives a0 = 1.
1.6. SPECIAL INTEGRAL 57
Solved Problem 1.50 Find the constant value of Fourier series integration of
the given function f(x) = x2
within limits from 0 to π.
Solution Constant term a0 in the Fourier series integration is given by
a0 =
1
b − a
b
Z
a
f(x) dx
Taking limits from 0 to π and function f(x) = x2
, the Fourier series constant
integral term becomes to
a0 =
1
π − 0
π
Z
0
x2
dx
Or
a0 =
1
π

x3
3
π
0
It gives a0 = π2
/3.
1.6.3 Specific Workout
In the following section, we do and find the definite integral by applying
conditions and will analysis our answers.
Solved Problem 1.51 Find the definite integration of f(x) = sin x within
limits from 0 to π/2. What would the answer if function is multiplied by x?
Solution The definite integral as per given problem is
I =
π/2
Z
0
sin x dx = [− cos x]π/2
0 = [− cos(π/2)] − [− cos 0]
This gives I = 1. Now, function is multiplied by x. So, new integral be
I′
=
π/2
Z
0
x × sin x dx = [x × − cos x]π/2
0 −
π/2
Z
0

d
dx
x
 Z
sin x dx

dx
58 Definite Integration
On simplification
I′
=
hπ
2
× − cos(π/2)
i
− [0 × − cos 0] −
π/2
Z
0
[− cos x] dx
Or
I′
= [sin x]π/2
0
It gives, I′
= 1. Thus both results are same.
Solved Problem 1.52 Find the definite integration of f(x) = cos x within
limits from 0 to π/2. What would the answer if function is multiplied by x?
Solution The definite integral as per given problem is
I =
π/2
Z
0
cos x dx = [sin x]π/2
0 = [sin(π/2)] − [sin 0]
This gives I = 1. Now, function is multiplied by x. So, new integral be
I′
=
π/2
Z
0
x × cos x dx = [x × sin x]π/2
0 −
π/2
Z
0

d
dx
x
 Z
cos x dx

dx
On simplification
I′
=
hπ
2
× sin(π/2)
i
− [0 × sin 0] −
π/2
Z
0
[sin x] dx
Or
I′
=
π
2
− 0 − [− cos x]π/2
0 =
π
2
+ cos(π/2) − cos 0
It gives, I′
= π
2
− 1. Thus both results are not same.
Solved Problem 1.53 Find the definite integration of f(x) = 1
1+x2 within
limits from 0 to ∞. What would the answer if function is multiplied by x?
Solution The definite integral as per given problem is
I =
∞
Z
0
1
1 + x2
dx =

tan−1
x
∞
0
=

tan−1
∞

−

tan−1
0
1.6. SPECIAL INTEGRAL 59
This gives I = π/2. Now, function is multiplied by x. So, new integral be
I′
=
∞
Z
0
x
1 + x2
dx
Put 1 + x2
= t. On differentiation, it gives, 2x dx = dt. Or it gives x dx =
dt/2. The corresponding limits are t = 1 to t = ∞ respectively when x = 0
and x = ∞. On simplification
I′
=
1
2
×
∞
Z
1
1
t
dt
Or
I′
=
1
2
× [ln t]∞
1
It gives that I′
is divergent in the given region of x. So, indefinite integral will
be found by substituting the value of t and neglecting the limits of integration
and adding a constant.
I′
=
ln(1 + x2
)
2
+ C
Solved Problem 1.54 Find the definite integration of f(k) = e−sk
within
limits from 0 to ∞. Here s is a constant. What would the answer if function
is multiplied by k?
Solution The definite integral as per given problem is
I =
∞
Z
0
e−sk
dk =
1
−s
×

e−sk
x
∞
0
=
1
−s
×

e−s×∞
− e−s×0

If s  0 then e−s×∞
will be equal to zero. If s  0 say s = −ǫ then e−s×∞
will be equal to eǫ×∞
and it will be equal to ∞. Second case is not acceptable
as integral in divergent, so taking first case (s  0) only, we have I = 1/s.
Now, function is multiplied by k. So, new integral be
I′
=
∞
Z
0
ke−sk
dk
60 Definite Integration
Or
I′
=

−
(sk + 1) e−sk
s2
∞
0
It gives I′
= 1/s2
.
1.6.4 Repetitive Integrals
Some function are periodic function in which their original value can be found
after two or more times integration. These types of functions can be solved
by assuming integral as I. f(x) and g(x) are two functions which returns
to itself on twice derivative or twice integral. Here, two functions, f(x) and
g(x) are taken. Derivating twice to the functions f(x) and g(x), they gave
d
dx
f(x) = f
′
(x);
d
dx
f
′
(x) = f(x)
d
dx
g(x) = g′
(x);
d
dx
g′
(x) = g(x)
Integrating twice to the same functions f(x) and g(x), we got
Z
f(x) dx = F(x);
Z
F(x) dx = f(x)
Z
g(x) dx = G(x);
Z
G(x) dx = g(x)
Now, let I is integral solution of integral of product of these two functions,
i.e. f(x) and g(x) about x. Now
I =
Z
f(x) × g(x) dx (1.41)
Now applying product rule of integration, we get
I = f(x)
Z
g(x) dx ±
Z 
d
dx
f(x)
Z
g(x) dx

dx (1.42)
On solving
I = f(x) G(x) ±
Z h
f
′
(x) G(x)
i
dx (1.43)
1.6. SPECIAL INTEGRAL 61
Again integrating second part of above relation
I = f(x) G(x)±

f
′
(x)
Z
G(x) dx ±
Z 
d
dx
f
′
(x) ×
Z
G(x) dx

dx

(1.44)
on solving
I = f(x) G(x) ±

f
′
(x) g(x) ±
Z
f(x) × g(x) dx

(1.45)
R
f(x) × g(x) dx is equal to I, hence above equation can be written as
I = f(x) G(x) ± f
′
(x) g(x) ± I (1.46)
Here, sign of I in right hand side is determinant. It may cancel to I in left
side or may added to it. Let the coefficient of I is k. So,
kI = f(x) G(x) ± f
′
(x) g(x) (1.47)
Here k is constant and integral solution I can be obtained by dividing right
hand side by k. ± is used to describe method of finding integral step by step.
Next is the solved problem that explains this method step by step.
Solved Problem 1.55 Find the integral of ex
sin(x).
Solution Let
I =
Z
ex
sin(x) dx
Using product rule, we have
I = ex
Z
sin(x) dx −
Z 
d
dx
ex
×
Z
sin(x) dx

dx
On simplification, we have
I = ex
× − cos(x) −
Z
[ex
× − cos(x)] dx
Again solving the second part of right hand side:
I = −ex
cos(x) +

ex
Z
cos(x) dx −
Z 
d
dx
ex
Z
cos x dx

dx
62 Definite Integration
On simplification
I = −ex
cos(x) + ex
sin(x) −
Z
ex
sin x dx
Or
2I = ex
(sin x − cos x)
Or
I =
ex
2
(sin x − cos x)
This is required result.
1.6.5 Derivative of Integrals
Let a continuous and differentiable function f(x) whose anti-derivative is
F(x) and derivative of F(x) is f(x) within the definite limits from a to b.
Here a  b are the function of t. Now the integral is
b
Z
a
f(x) dx = F(a) − F(b)
Now derivative of the given function with respect to t is
d
dt
b
Z
a
f(x) dx =
d
dt
[F(b) − F(a)]
= f(b) ·
d
dt
b − f(a) ·
d
dt
a
Or
d
dt


b
Z
a
f(x) dx

 = f(b) ·
db
dt
− f(a) ·
da
dt
(1.48)
This is the derivative of a function under the sign of definite integral.
Solved Problem 1.56 Evaluate the relation
d
dx
x2
Z
0
sin(t) dt
1.6. SPECIAL INTEGRAL 63
Solution Now from the relation
d
dt


b
Z
a
f(x) dx

 = f(b) ·
db
dt
− f(a) ·
da
dt
Here function f(t) = sin(t). The result of the above relation will be
d
dx
x2
Z
0
sin(t) dt = f(x2
) ·
d
dx
x2
− f(0) ·
d
dt
0
= 2x sin(x2
) − 0
= 2x sin(x2
)
It is required result.
Solved Problem 1.57 Evaluate
d
dx
x2
Z
0
p
sin(t) + cos(t) dt
Solution Now from the relation
d
dt


b
Z
a
f(x) dx

 = f(b) ·
db
dt
− f(a) ·
da
dt
Here function f(t) =
p
sin(t) + cos(t). The result of the above relation will
be
d
dx
x2
Z
0
p
sin(t) + cos(t) dt = f(x2
) ·
d
dx
x2
− f(0) ·
d
dt
0
= 2x
p
sin(x2) + cos(x2) − 0
= 2x
p
sin(x2) + cos(x2)
It is required result.

More Related Content

What's hot

application of complex numbers
application of complex numbersapplication of complex numbers
application of complex numbersKaustubh Garud
 
Maxima & Minima of Functions - Differential Calculus by Arun Umrao
Maxima & Minima of Functions - Differential Calculus by Arun UmraoMaxima & Minima of Functions - Differential Calculus by Arun Umrao
Maxima & Minima of Functions - Differential Calculus by Arun Umraossuserd6b1fd
 
Principle of Derivative Calculus - Differential Calculus - An Introduction by...
Principle of Derivative Calculus - Differential Calculus - An Introduction by...Principle of Derivative Calculus - Differential Calculus - An Introduction by...
Principle of Derivative Calculus - Differential Calculus - An Introduction by...ssuserd6b1fd
 
27 power series x
27 power series x27 power series x
27 power series xmath266
 
Applied mathematics I for enginering
Applied mathematics  I for engineringApplied mathematics  I for enginering
Applied mathematics I for engineringgeletameyu
 
Matlab lab manual
Matlab lab manualMatlab lab manual
Matlab lab manualnmahi96
 
Decreasing and increasing functions by arun umrao
Decreasing and increasing functions by arun umraoDecreasing and increasing functions by arun umrao
Decreasing and increasing functions by arun umraossuserd6b1fd
 
Deturck wilf
Deturck wilfDeturck wilf
Deturck wilfCAALAAA
 
FEM problem of elasticity
FEM problem of elasticityFEM problem of elasticity
FEM problem of elasticityAshwani Jha
 

What's hot (14)

application of complex numbers
application of complex numbersapplication of complex numbers
application of complex numbers
 
Maxima & Minima of Functions - Differential Calculus by Arun Umrao
Maxima & Minima of Functions - Differential Calculus by Arun UmraoMaxima & Minima of Functions - Differential Calculus by Arun Umrao
Maxima & Minima of Functions - Differential Calculus by Arun Umrao
 
COMPLEX NUMBER
COMPLEX NUMBERCOMPLEX NUMBER
COMPLEX NUMBER
 
Principle of Derivative Calculus - Differential Calculus - An Introduction by...
Principle of Derivative Calculus - Differential Calculus - An Introduction by...Principle of Derivative Calculus - Differential Calculus - An Introduction by...
Principle of Derivative Calculus - Differential Calculus - An Introduction by...
 
27 power series x
27 power series x27 power series x
27 power series x
 
Applied mathematics I for enginering
Applied mathematics  I for engineringApplied mathematics  I for enginering
Applied mathematics I for enginering
 
Matlab lab manual
Matlab lab manualMatlab lab manual
Matlab lab manual
 
Decreasing and increasing functions by arun umrao
Decreasing and increasing functions by arun umraoDecreasing and increasing functions by arun umrao
Decreasing and increasing functions by arun umrao
 
Deturck wilf
Deturck wilfDeturck wilf
Deturck wilf
 
Operations Research - The Dual Simplex Method
Operations Research - The Dual Simplex MethodOperations Research - The Dual Simplex Method
Operations Research - The Dual Simplex Method
 
Web_Alg_Project
Web_Alg_ProjectWeb_Alg_Project
Web_Alg_Project
 
presentazione
presentazionepresentazione
presentazione
 
Duality
DualityDuality
Duality
 
FEM problem of elasticity
FEM problem of elasticityFEM problem of elasticity
FEM problem of elasticity
 

Similar to Principle of Definite Integra - Integral Calculus - by Arun Umrao

Function Analysis v.1
Function Analysis v.1Function Analysis v.1
Function Analysis v.1Arun Umrao
 
Integration material
Integration material Integration material
Integration material Surya Swaroop
 
Average value by integral method
Average value by integral methodAverage value by integral method
Average value by integral methodArun Umrao
 
01. Functions-Theory & Solved Examples Module-4.pdf
01. Functions-Theory & Solved Examples Module-4.pdf01. Functions-Theory & Solved Examples Module-4.pdf
01. Functions-Theory & Solved Examples Module-4.pdfRajuSingh806014
 
Opt simple single_000
Opt simple single_000Opt simple single_000
Opt simple single_000sheetslibrary
 
The Fundamental theorem of calculus
The Fundamental theorem of calculus The Fundamental theorem of calculus
The Fundamental theorem of calculus AhsanIrshad8
 
L2 graphs piecewise, absolute,and greatest integer
L2 graphs  piecewise, absolute,and greatest integerL2 graphs  piecewise, absolute,and greatest integer
L2 graphs piecewise, absolute,and greatest integerJames Tagara
 
Basics of Integration and Derivatives
Basics of Integration and DerivativesBasics of Integration and Derivatives
Basics of Integration and DerivativesFaisal Waqar
 
7.curves Further Mathematics Zimbabwe Zimsec Cambridge
7.curves   Further Mathematics Zimbabwe Zimsec Cambridge7.curves   Further Mathematics Zimbabwe Zimsec Cambridge
7.curves Further Mathematics Zimbabwe Zimsec Cambridgealproelearning
 
Chapter 1 (math 1)
Chapter 1 (math 1)Chapter 1 (math 1)
Chapter 1 (math 1)Amr Mohamed
 
Add Math(F4) Quadratic Function 3.1
Add Math(F4)  Quadratic Function  3.1Add Math(F4)  Quadratic Function  3.1
Add Math(F4) Quadratic Function 3.1roszelan
 
Module 3 exponential and logarithmic functions
Module 3   exponential and logarithmic functionsModule 3   exponential and logarithmic functions
Module 3 exponential and logarithmic functionsdionesioable
 
monotonicity thoery & solved & execise Module-4.pdf
monotonicity thoery & solved & execise Module-4.pdfmonotonicity thoery & solved & execise Module-4.pdf
monotonicity thoery & solved & execise Module-4.pdfRajuSingh806014
 
Limit & continuity
Limit & continuityLimit & continuity
Limit & continuityArun Umrao
 

Similar to Principle of Definite Integra - Integral Calculus - by Arun Umrao (20)

Chap07
Chap07Chap07
Chap07
 
Function Analysis v.1
Function Analysis v.1Function Analysis v.1
Function Analysis v.1
 
Integration material
Integration material Integration material
Integration material
 
Integration
IntegrationIntegration
Integration
 
Average value by integral method
Average value by integral methodAverage value by integral method
Average value by integral method
 
Functions
FunctionsFunctions
Functions
 
Graph a function
Graph a functionGraph a function
Graph a function
 
Functions (Theory)
Functions (Theory)Functions (Theory)
Functions (Theory)
 
01. Functions-Theory & Solved Examples Module-4.pdf
01. Functions-Theory & Solved Examples Module-4.pdf01. Functions-Theory & Solved Examples Module-4.pdf
01. Functions-Theory & Solved Examples Module-4.pdf
 
The integral
The integralThe integral
The integral
 
Opt simple single_000
Opt simple single_000Opt simple single_000
Opt simple single_000
 
The Fundamental theorem of calculus
The Fundamental theorem of calculus The Fundamental theorem of calculus
The Fundamental theorem of calculus
 
L2 graphs piecewise, absolute,and greatest integer
L2 graphs  piecewise, absolute,and greatest integerL2 graphs  piecewise, absolute,and greatest integer
L2 graphs piecewise, absolute,and greatest integer
 
Basics of Integration and Derivatives
Basics of Integration and DerivativesBasics of Integration and Derivatives
Basics of Integration and Derivatives
 
7.curves Further Mathematics Zimbabwe Zimsec Cambridge
7.curves   Further Mathematics Zimbabwe Zimsec Cambridge7.curves   Further Mathematics Zimbabwe Zimsec Cambridge
7.curves Further Mathematics Zimbabwe Zimsec Cambridge
 
Chapter 1 (math 1)
Chapter 1 (math 1)Chapter 1 (math 1)
Chapter 1 (math 1)
 
Add Math(F4) Quadratic Function 3.1
Add Math(F4)  Quadratic Function  3.1Add Math(F4)  Quadratic Function  3.1
Add Math(F4) Quadratic Function 3.1
 
Module 3 exponential and logarithmic functions
Module 3   exponential and logarithmic functionsModule 3   exponential and logarithmic functions
Module 3 exponential and logarithmic functions
 
monotonicity thoery & solved & execise Module-4.pdf
monotonicity thoery & solved & execise Module-4.pdfmonotonicity thoery & solved & execise Module-4.pdf
monotonicity thoery & solved & execise Module-4.pdf
 
Limit & continuity
Limit & continuityLimit & continuity
Limit & continuity
 

More from ssuserd6b1fd

Notes for c programming for mca, bca, b. tech cse, ece and msc (cs) 1 of 5 by...
Notes for c programming for mca, bca, b. tech cse, ece and msc (cs) 1 of 5 by...Notes for c programming for mca, bca, b. tech cse, ece and msc (cs) 1 of 5 by...
Notes for c programming for mca, bca, b. tech cse, ece and msc (cs) 1 of 5 by...ssuserd6b1fd
 
Decreasing increasing functions by arun umrao
Decreasing increasing functions by arun umraoDecreasing increasing functions by arun umrao
Decreasing increasing functions by arun umraossuserd6b1fd
 
Distribution of normal data understanding it numerical way by arun umrao
Distribution of normal data   understanding it numerical way by arun umraoDistribution of normal data   understanding it numerical way by arun umrao
Distribution of normal data understanding it numerical way by arun umraossuserd6b1fd
 
What is meaning of epsilon and delta in limits of a function by Arun Umrao
What is meaning of epsilon and delta in limits of a function by Arun UmraoWhat is meaning of epsilon and delta in limits of a function by Arun Umrao
What is meaning of epsilon and delta in limits of a function by Arun Umraossuserd6b1fd
 
Notes for GNU Octave - Numerical Programming - for Students - 02 of 02 by aru...
Notes for GNU Octave - Numerical Programming - for Students - 02 of 02 by aru...Notes for GNU Octave - Numerical Programming - for Students - 02 of 02 by aru...
Notes for GNU Octave - Numerical Programming - for Students - 02 of 02 by aru...ssuserd6b1fd
 
Notes for GNU Octave - Numerical Programming - for Students 01 of 02 by Arun ...
Notes for GNU Octave - Numerical Programming - for Students 01 of 02 by Arun ...Notes for GNU Octave - Numerical Programming - for Students 01 of 02 by Arun ...
Notes for GNU Octave - Numerical Programming - for Students 01 of 02 by Arun ...ssuserd6b1fd
 
Notes for C++ Programming / Object Oriented C++ Programming for MCA, BCA and ...
Notes for C++ Programming / Object Oriented C++ Programming for MCA, BCA and ...Notes for C++ Programming / Object Oriented C++ Programming for MCA, BCA and ...
Notes for C++ Programming / Object Oriented C++ Programming for MCA, BCA and ...ssuserd6b1fd
 
Notes for C++ Programming / Object Oriented C++ Programming for MCA, BCA and ...
Notes for C++ Programming / Object Oriented C++ Programming for MCA, BCA and ...Notes for C++ Programming / Object Oriented C++ Programming for MCA, BCA and ...
Notes for C++ Programming / Object Oriented C++ Programming for MCA, BCA and ...ssuserd6b1fd
 
Think Like Scilab and Become a Numerical Programming Expert- Notes for Beginn...
Think Like Scilab and Become a Numerical Programming Expert- Notes for Beginn...Think Like Scilab and Become a Numerical Programming Expert- Notes for Beginn...
Think Like Scilab and Become a Numerical Programming Expert- Notes for Beginn...ssuserd6b1fd
 
Notes for C Programming for MCA, BCA, B. Tech CSE, ECE and MSC (CS) 5 of 5 by...
Notes for C Programming for MCA, BCA, B. Tech CSE, ECE and MSC (CS) 5 of 5 by...Notes for C Programming for MCA, BCA, B. Tech CSE, ECE and MSC (CS) 5 of 5 by...
Notes for C Programming for MCA, BCA, B. Tech CSE, ECE and MSC (CS) 5 of 5 by...ssuserd6b1fd
 
Notes for C Programming for MCA, BCA, B. Tech CSE, ECE and MSC (CS) 4 of 5 by...
Notes for C Programming for MCA, BCA, B. Tech CSE, ECE and MSC (CS) 4 of 5 by...Notes for C Programming for MCA, BCA, B. Tech CSE, ECE and MSC (CS) 4 of 5 by...
Notes for C Programming for MCA, BCA, B. Tech CSE, ECE and MSC (CS) 4 of 5 by...ssuserd6b1fd
 
Notes for C Programming for MCA, BCA, B. Tech CSE, ECE and MSC (CS) 3 of 5 b...
Notes for C Programming for MCA, BCA, B. Tech CSE, ECE and MSC (CS) 3 of 5  b...Notes for C Programming for MCA, BCA, B. Tech CSE, ECE and MSC (CS) 3 of 5  b...
Notes for C Programming for MCA, BCA, B. Tech CSE, ECE and MSC (CS) 3 of 5 b...ssuserd6b1fd
 
Notes for C Programming for MCA, BCA, B. Tech CSE, ECE and MSC (CS) 2 of 5 by...
Notes for C Programming for MCA, BCA, B. Tech CSE, ECE and MSC (CS) 2 of 5 by...Notes for C Programming for MCA, BCA, B. Tech CSE, ECE and MSC (CS) 2 of 5 by...
Notes for C Programming for MCA, BCA, B. Tech CSE, ECE and MSC (CS) 2 of 5 by...ssuserd6b1fd
 
Notes for C Programming for MCA, BCA, B. Tech CSE, ECE and MSC (CS) 1 of 5 by...
Notes for C Programming for MCA, BCA, B. Tech CSE, ECE and MSC (CS) 1 of 5 by...Notes for C Programming for MCA, BCA, B. Tech CSE, ECE and MSC (CS) 1 of 5 by...
Notes for C Programming for MCA, BCA, B. Tech CSE, ECE and MSC (CS) 1 of 5 by...ssuserd6b1fd
 
Notes and Description for Xcos Scilab Block Simulation with Suitable Examples...
Notes and Description for Xcos Scilab Block Simulation with Suitable Examples...Notes and Description for Xcos Scilab Block Simulation with Suitable Examples...
Notes and Description for Xcos Scilab Block Simulation with Suitable Examples...ssuserd6b1fd
 
Work and Energy Notes by Arun Umrao
Work and Energy Notes by Arun UmraoWork and Energy Notes by Arun Umrao
Work and Energy Notes by Arun Umraossuserd6b1fd
 
Physics dictionary for CBSE, ISCE, Class X Students by Arun Umrao
Physics dictionary for CBSE, ISCE, Class X Students by Arun UmraoPhysics dictionary for CBSE, ISCE, Class X Students by Arun Umrao
Physics dictionary for CBSE, ISCE, Class X Students by Arun Umraossuserd6b1fd
 
Java Programming Notes for Beginners by Arun Umrao
Java Programming Notes for Beginners by Arun UmraoJava Programming Notes for Beginners by Arun Umrao
Java Programming Notes for Beginners by Arun Umraossuserd6b1fd
 
Notes of 8085 micro processor Programming for BCA, MCA, MSC (CS), MSC (IT) &...
Notes of 8085 micro processor Programming  for BCA, MCA, MSC (CS), MSC (IT) &...Notes of 8085 micro processor Programming  for BCA, MCA, MSC (CS), MSC (IT) &...
Notes of 8085 micro processor Programming for BCA, MCA, MSC (CS), MSC (IT) &...ssuserd6b1fd
 
Notes of 8051 Micro Controller for BCA, MCA, MSC (CS), MSC (IT) & AMIE IEI- b...
Notes of 8051 Micro Controller for BCA, MCA, MSC (CS), MSC (IT) & AMIE IEI- b...Notes of 8051 Micro Controller for BCA, MCA, MSC (CS), MSC (IT) & AMIE IEI- b...
Notes of 8051 Micro Controller for BCA, MCA, MSC (CS), MSC (IT) & AMIE IEI- b...ssuserd6b1fd
 

More from ssuserd6b1fd (20)

Notes for c programming for mca, bca, b. tech cse, ece and msc (cs) 1 of 5 by...
Notes for c programming for mca, bca, b. tech cse, ece and msc (cs) 1 of 5 by...Notes for c programming for mca, bca, b. tech cse, ece and msc (cs) 1 of 5 by...
Notes for c programming for mca, bca, b. tech cse, ece and msc (cs) 1 of 5 by...
 
Decreasing increasing functions by arun umrao
Decreasing increasing functions by arun umraoDecreasing increasing functions by arun umrao
Decreasing increasing functions by arun umrao
 
Distribution of normal data understanding it numerical way by arun umrao
Distribution of normal data   understanding it numerical way by arun umraoDistribution of normal data   understanding it numerical way by arun umrao
Distribution of normal data understanding it numerical way by arun umrao
 
What is meaning of epsilon and delta in limits of a function by Arun Umrao
What is meaning of epsilon and delta in limits of a function by Arun UmraoWhat is meaning of epsilon and delta in limits of a function by Arun Umrao
What is meaning of epsilon and delta in limits of a function by Arun Umrao
 
Notes for GNU Octave - Numerical Programming - for Students - 02 of 02 by aru...
Notes for GNU Octave - Numerical Programming - for Students - 02 of 02 by aru...Notes for GNU Octave - Numerical Programming - for Students - 02 of 02 by aru...
Notes for GNU Octave - Numerical Programming - for Students - 02 of 02 by aru...
 
Notes for GNU Octave - Numerical Programming - for Students 01 of 02 by Arun ...
Notes for GNU Octave - Numerical Programming - for Students 01 of 02 by Arun ...Notes for GNU Octave - Numerical Programming - for Students 01 of 02 by Arun ...
Notes for GNU Octave - Numerical Programming - for Students 01 of 02 by Arun ...
 
Notes for C++ Programming / Object Oriented C++ Programming for MCA, BCA and ...
Notes for C++ Programming / Object Oriented C++ Programming for MCA, BCA and ...Notes for C++ Programming / Object Oriented C++ Programming for MCA, BCA and ...
Notes for C++ Programming / Object Oriented C++ Programming for MCA, BCA and ...
 
Notes for C++ Programming / Object Oriented C++ Programming for MCA, BCA and ...
Notes for C++ Programming / Object Oriented C++ Programming for MCA, BCA and ...Notes for C++ Programming / Object Oriented C++ Programming for MCA, BCA and ...
Notes for C++ Programming / Object Oriented C++ Programming for MCA, BCA and ...
 
Think Like Scilab and Become a Numerical Programming Expert- Notes for Beginn...
Think Like Scilab and Become a Numerical Programming Expert- Notes for Beginn...Think Like Scilab and Become a Numerical Programming Expert- Notes for Beginn...
Think Like Scilab and Become a Numerical Programming Expert- Notes for Beginn...
 
Notes for C Programming for MCA, BCA, B. Tech CSE, ECE and MSC (CS) 5 of 5 by...
Notes for C Programming for MCA, BCA, B. Tech CSE, ECE and MSC (CS) 5 of 5 by...Notes for C Programming for MCA, BCA, B. Tech CSE, ECE and MSC (CS) 5 of 5 by...
Notes for C Programming for MCA, BCA, B. Tech CSE, ECE and MSC (CS) 5 of 5 by...
 
Notes for C Programming for MCA, BCA, B. Tech CSE, ECE and MSC (CS) 4 of 5 by...
Notes for C Programming for MCA, BCA, B. Tech CSE, ECE and MSC (CS) 4 of 5 by...Notes for C Programming for MCA, BCA, B. Tech CSE, ECE and MSC (CS) 4 of 5 by...
Notes for C Programming for MCA, BCA, B. Tech CSE, ECE and MSC (CS) 4 of 5 by...
 
Notes for C Programming for MCA, BCA, B. Tech CSE, ECE and MSC (CS) 3 of 5 b...
Notes for C Programming for MCA, BCA, B. Tech CSE, ECE and MSC (CS) 3 of 5  b...Notes for C Programming for MCA, BCA, B. Tech CSE, ECE and MSC (CS) 3 of 5  b...
Notes for C Programming for MCA, BCA, B. Tech CSE, ECE and MSC (CS) 3 of 5 b...
 
Notes for C Programming for MCA, BCA, B. Tech CSE, ECE and MSC (CS) 2 of 5 by...
Notes for C Programming for MCA, BCA, B. Tech CSE, ECE and MSC (CS) 2 of 5 by...Notes for C Programming for MCA, BCA, B. Tech CSE, ECE and MSC (CS) 2 of 5 by...
Notes for C Programming for MCA, BCA, B. Tech CSE, ECE and MSC (CS) 2 of 5 by...
 
Notes for C Programming for MCA, BCA, B. Tech CSE, ECE and MSC (CS) 1 of 5 by...
Notes for C Programming for MCA, BCA, B. Tech CSE, ECE and MSC (CS) 1 of 5 by...Notes for C Programming for MCA, BCA, B. Tech CSE, ECE and MSC (CS) 1 of 5 by...
Notes for C Programming for MCA, BCA, B. Tech CSE, ECE and MSC (CS) 1 of 5 by...
 
Notes and Description for Xcos Scilab Block Simulation with Suitable Examples...
Notes and Description for Xcos Scilab Block Simulation with Suitable Examples...Notes and Description for Xcos Scilab Block Simulation with Suitable Examples...
Notes and Description for Xcos Scilab Block Simulation with Suitable Examples...
 
Work and Energy Notes by Arun Umrao
Work and Energy Notes by Arun UmraoWork and Energy Notes by Arun Umrao
Work and Energy Notes by Arun Umrao
 
Physics dictionary for CBSE, ISCE, Class X Students by Arun Umrao
Physics dictionary for CBSE, ISCE, Class X Students by Arun UmraoPhysics dictionary for CBSE, ISCE, Class X Students by Arun Umrao
Physics dictionary for CBSE, ISCE, Class X Students by Arun Umrao
 
Java Programming Notes for Beginners by Arun Umrao
Java Programming Notes for Beginners by Arun UmraoJava Programming Notes for Beginners by Arun Umrao
Java Programming Notes for Beginners by Arun Umrao
 
Notes of 8085 micro processor Programming for BCA, MCA, MSC (CS), MSC (IT) &...
Notes of 8085 micro processor Programming  for BCA, MCA, MSC (CS), MSC (IT) &...Notes of 8085 micro processor Programming  for BCA, MCA, MSC (CS), MSC (IT) &...
Notes of 8085 micro processor Programming for BCA, MCA, MSC (CS), MSC (IT) &...
 
Notes of 8051 Micro Controller for BCA, MCA, MSC (CS), MSC (IT) & AMIE IEI- b...
Notes of 8051 Micro Controller for BCA, MCA, MSC (CS), MSC (IT) & AMIE IEI- b...Notes of 8051 Micro Controller for BCA, MCA, MSC (CS), MSC (IT) & AMIE IEI- b...
Notes of 8051 Micro Controller for BCA, MCA, MSC (CS), MSC (IT) & AMIE IEI- b...
 

Recently uploaded

How to Make a Pirate ship Primary Education.pptx
How to Make a Pirate ship Primary Education.pptxHow to Make a Pirate ship Primary Education.pptx
How to Make a Pirate ship Primary Education.pptxmanuelaromero2013
 
Final demo Grade 9 for demo Plan dessert.pptx
Final demo Grade 9 for demo Plan dessert.pptxFinal demo Grade 9 for demo Plan dessert.pptx
Final demo Grade 9 for demo Plan dessert.pptxAvyJaneVismanos
 
CELL CYCLE Division Science 8 quarter IV.pptx
CELL CYCLE Division Science 8 quarter IV.pptxCELL CYCLE Division Science 8 quarter IV.pptx
CELL CYCLE Division Science 8 quarter IV.pptxJiesonDelaCerna
 
Presiding Officer Training module 2024 lok sabha elections
Presiding Officer Training module 2024 lok sabha electionsPresiding Officer Training module 2024 lok sabha elections
Presiding Officer Training module 2024 lok sabha electionsanshu789521
 
EPANDING THE CONTENT OF AN OUTLINE using notes.pptx
EPANDING THE CONTENT OF AN OUTLINE using notes.pptxEPANDING THE CONTENT OF AN OUTLINE using notes.pptx
EPANDING THE CONTENT OF AN OUTLINE using notes.pptxRaymartEstabillo3
 
AmericanHighSchoolsprezentacijaoskolama.
AmericanHighSchoolsprezentacijaoskolama.AmericanHighSchoolsprezentacijaoskolama.
AmericanHighSchoolsprezentacijaoskolama.arsicmarija21
 
Introduction to AI in Higher Education_draft.pptx
Introduction to AI in Higher Education_draft.pptxIntroduction to AI in Higher Education_draft.pptx
Introduction to AI in Higher Education_draft.pptxpboyjonauth
 
Proudly South Africa powerpoint Thorisha.pptx
Proudly South Africa powerpoint Thorisha.pptxProudly South Africa powerpoint Thorisha.pptx
Proudly South Africa powerpoint Thorisha.pptxthorishapillay1
 
Introduction to ArtificiaI Intelligence in Higher Education
Introduction to ArtificiaI Intelligence in Higher EducationIntroduction to ArtificiaI Intelligence in Higher Education
Introduction to ArtificiaI Intelligence in Higher Educationpboyjonauth
 
Earth Day Presentation wow hello nice great
Earth Day Presentation wow hello nice greatEarth Day Presentation wow hello nice great
Earth Day Presentation wow hello nice greatYousafMalik24
 
MICROBIOLOGY biochemical test detailed.pptx
MICROBIOLOGY biochemical test detailed.pptxMICROBIOLOGY biochemical test detailed.pptx
MICROBIOLOGY biochemical test detailed.pptxabhijeetpadhi001
 
Gas measurement O2,Co2,& ph) 04/2024.pptx
Gas measurement O2,Co2,& ph) 04/2024.pptxGas measurement O2,Co2,& ph) 04/2024.pptx
Gas measurement O2,Co2,& ph) 04/2024.pptxDr.Ibrahim Hassaan
 
Difference Between Search & Browse Methods in Odoo 17
Difference Between Search & Browse Methods in Odoo 17Difference Between Search & Browse Methods in Odoo 17
Difference Between Search & Browse Methods in Odoo 17Celine George
 
Employee wellbeing at the workplace.pptx
Employee wellbeing at the workplace.pptxEmployee wellbeing at the workplace.pptx
Employee wellbeing at the workplace.pptxNirmalaLoungPoorunde1
 
“Oh GOSH! Reflecting on Hackteria's Collaborative Practices in a Global Do-It...
“Oh GOSH! Reflecting on Hackteria's Collaborative Practices in a Global Do-It...“Oh GOSH! Reflecting on Hackteria's Collaborative Practices in a Global Do-It...
“Oh GOSH! Reflecting on Hackteria's Collaborative Practices in a Global Do-It...Marc Dusseiller Dusjagr
 
POINT- BIOCHEMISTRY SEM 2 ENZYMES UNIT 5.pptx
POINT- BIOCHEMISTRY SEM 2 ENZYMES UNIT 5.pptxPOINT- BIOCHEMISTRY SEM 2 ENZYMES UNIT 5.pptx
POINT- BIOCHEMISTRY SEM 2 ENZYMES UNIT 5.pptxSayali Powar
 
MARGINALIZATION (Different learners in Marginalized Group
MARGINALIZATION (Different learners in Marginalized GroupMARGINALIZATION (Different learners in Marginalized Group
MARGINALIZATION (Different learners in Marginalized GroupJonathanParaisoCruz
 
Full Stack Web Development Course for Beginners
Full Stack Web Development Course  for BeginnersFull Stack Web Development Course  for Beginners
Full Stack Web Development Course for BeginnersSabitha Banu
 
ENGLISH 7_Q4_LESSON 2_ Employing a Variety of Strategies for Effective Interp...
ENGLISH 7_Q4_LESSON 2_ Employing a Variety of Strategies for Effective Interp...ENGLISH 7_Q4_LESSON 2_ Employing a Variety of Strategies for Effective Interp...
ENGLISH 7_Q4_LESSON 2_ Employing a Variety of Strategies for Effective Interp...JhezDiaz1
 
Crayon Activity Handout For the Crayon A
Crayon Activity Handout For the Crayon ACrayon Activity Handout For the Crayon A
Crayon Activity Handout For the Crayon AUnboundStockton
 

Recently uploaded (20)

How to Make a Pirate ship Primary Education.pptx
How to Make a Pirate ship Primary Education.pptxHow to Make a Pirate ship Primary Education.pptx
How to Make a Pirate ship Primary Education.pptx
 
Final demo Grade 9 for demo Plan dessert.pptx
Final demo Grade 9 for demo Plan dessert.pptxFinal demo Grade 9 for demo Plan dessert.pptx
Final demo Grade 9 for demo Plan dessert.pptx
 
CELL CYCLE Division Science 8 quarter IV.pptx
CELL CYCLE Division Science 8 quarter IV.pptxCELL CYCLE Division Science 8 quarter IV.pptx
CELL CYCLE Division Science 8 quarter IV.pptx
 
Presiding Officer Training module 2024 lok sabha elections
Presiding Officer Training module 2024 lok sabha electionsPresiding Officer Training module 2024 lok sabha elections
Presiding Officer Training module 2024 lok sabha elections
 
EPANDING THE CONTENT OF AN OUTLINE using notes.pptx
EPANDING THE CONTENT OF AN OUTLINE using notes.pptxEPANDING THE CONTENT OF AN OUTLINE using notes.pptx
EPANDING THE CONTENT OF AN OUTLINE using notes.pptx
 
AmericanHighSchoolsprezentacijaoskolama.
AmericanHighSchoolsprezentacijaoskolama.AmericanHighSchoolsprezentacijaoskolama.
AmericanHighSchoolsprezentacijaoskolama.
 
Introduction to AI in Higher Education_draft.pptx
Introduction to AI in Higher Education_draft.pptxIntroduction to AI in Higher Education_draft.pptx
Introduction to AI in Higher Education_draft.pptx
 
Proudly South Africa powerpoint Thorisha.pptx
Proudly South Africa powerpoint Thorisha.pptxProudly South Africa powerpoint Thorisha.pptx
Proudly South Africa powerpoint Thorisha.pptx
 
Introduction to ArtificiaI Intelligence in Higher Education
Introduction to ArtificiaI Intelligence in Higher EducationIntroduction to ArtificiaI Intelligence in Higher Education
Introduction to ArtificiaI Intelligence in Higher Education
 
Earth Day Presentation wow hello nice great
Earth Day Presentation wow hello nice greatEarth Day Presentation wow hello nice great
Earth Day Presentation wow hello nice great
 
MICROBIOLOGY biochemical test detailed.pptx
MICROBIOLOGY biochemical test detailed.pptxMICROBIOLOGY biochemical test detailed.pptx
MICROBIOLOGY biochemical test detailed.pptx
 
Gas measurement O2,Co2,& ph) 04/2024.pptx
Gas measurement O2,Co2,& ph) 04/2024.pptxGas measurement O2,Co2,& ph) 04/2024.pptx
Gas measurement O2,Co2,& ph) 04/2024.pptx
 
Difference Between Search & Browse Methods in Odoo 17
Difference Between Search & Browse Methods in Odoo 17Difference Between Search & Browse Methods in Odoo 17
Difference Between Search & Browse Methods in Odoo 17
 
Employee wellbeing at the workplace.pptx
Employee wellbeing at the workplace.pptxEmployee wellbeing at the workplace.pptx
Employee wellbeing at the workplace.pptx
 
“Oh GOSH! Reflecting on Hackteria's Collaborative Practices in a Global Do-It...
“Oh GOSH! Reflecting on Hackteria's Collaborative Practices in a Global Do-It...“Oh GOSH! Reflecting on Hackteria's Collaborative Practices in a Global Do-It...
“Oh GOSH! Reflecting on Hackteria's Collaborative Practices in a Global Do-It...
 
POINT- BIOCHEMISTRY SEM 2 ENZYMES UNIT 5.pptx
POINT- BIOCHEMISTRY SEM 2 ENZYMES UNIT 5.pptxPOINT- BIOCHEMISTRY SEM 2 ENZYMES UNIT 5.pptx
POINT- BIOCHEMISTRY SEM 2 ENZYMES UNIT 5.pptx
 
MARGINALIZATION (Different learners in Marginalized Group
MARGINALIZATION (Different learners in Marginalized GroupMARGINALIZATION (Different learners in Marginalized Group
MARGINALIZATION (Different learners in Marginalized Group
 
Full Stack Web Development Course for Beginners
Full Stack Web Development Course  for BeginnersFull Stack Web Development Course  for Beginners
Full Stack Web Development Course for Beginners
 
ENGLISH 7_Q4_LESSON 2_ Employing a Variety of Strategies for Effective Interp...
ENGLISH 7_Q4_LESSON 2_ Employing a Variety of Strategies for Effective Interp...ENGLISH 7_Q4_LESSON 2_ Employing a Variety of Strategies for Effective Interp...
ENGLISH 7_Q4_LESSON 2_ Employing a Variety of Strategies for Effective Interp...
 
Crayon Activity Handout For the Crayon A
Crayon Activity Handout For the Crayon ACrayon Activity Handout For the Crayon A
Crayon Activity Handout For the Crayon A
 

Principle of Definite Integra - Integral Calculus - by Arun Umrao

  • 1. 1 DEFINITE INTEGRAL A SHORT NOTES Arun Umrao https://sites.google.com/view/arunumrao DRAFT COPY - GPL LICENSING
  • 2. 2 Definite Integration Contents 1 Definite Integration 3 1.1 Definite Integration . . . . . . . . . . . . . . . . . . . . . . . . 3 1.2 Properties . . . . . . . . . . . . . . . . . . . . . . . . . . . . . 6 1.2.1 In-dependent of Variable . . . . . . . . . . . . . . . . . 6 1.2.2 Flipping of Limits . . . . . . . . . . . . . . . . . . . . . 7 1.2.3 Splitting of Limits . . . . . . . . . . . . . . . . . . . . 8 1.2.4 Shifting of Variable . . . . . . . . . . . . . . . . . . . . 12 1.2.5 Shifting of Periodic Function . . . . . . . . . . . . . . . 14 1.2.6 Even & Odd Function . . . . . . . . . . . . . . . . . . 16 1.2.7 Periodic Even & Odd Function . . . . . . . . . . . . . 23 1.2.8 Splitting of Limit in n Parts . . . . . . . . . . . . . . . 25 1.2.9 Definite Integration & Limits . . . . . . . . . . . . . . 32 1.3 Average Value . . . . . . . . . . . . . . . . . . . . . . . . . . . 34 1.4 Piece-wise Functions . . . . . . . . . . . . . . . . . . . . . . . 37 1.5 Integral Identities . . . . . . . . . . . . . . . . . . . . . . . . . 47 1.6 Special Integral . . . . . . . . . . . . . . . . . . . . . . . . . . 50 1.6.1 Definite Integral of Laplace Type . . . . . . . . . . . . 50 1.6.2 Definite Integral of Fourier Type . . . . . . . . . . . . 54 1.6.3 Specific Workout . . . . . . . . . . . . . . . . . . . . . 57 1.6.4 Repetitive Integrals . . . . . . . . . . . . . . . . . . . . 60 1.6.5 Derivative of Integrals . . . . . . . . . . . . . . . . . . 62 1.6.6 Vector Integration . . . . . . . . . . . . . . . . . . . . 64 1.6.7 Principal & General Value . . . . . . . . . . . . . . . . 65 1.6.8 Integral of Graphs . . . . . . . . . . . . . . . . . . . . 66 1.6.9 Vertically symmetric Function . . . . . . . . . . . . . . 72 1.7 Exponential Trigonometric Functions . . . . . . . . . . . . . . 77
  • 3. 1.1. DEFINITE INTEGRATION 3 1Definite Integration In this chapter, the integration of infinite continuous function is bounded between two limits. 1.1 Definite Integration When integration of a function is limited between two fixed points where function is continuous and limit exists everywhere between these two limits, then The integral is called definite integration. If f(x) is a continuous func- tion in [a, b] and limit exists everywhere in [a, b] then definite integration of the function is given by I = b Z a f(x) dx (1.1) Definite integral depends on lower limit and upper limit only. It is indepen- dent of any intermediate values between [a, b]. If F(x) is integral of f(x) and there are intermediate points j, k and l such that [a, j], [j, k], [k, l], [l, b] then I = j Z a f(x) dx + k Z j f(x) dx + l Z k f(x) dx + b Z l f(x) dx and I = F(j) − F(a) + F(k) − F(j) + F(l) − F(k) + F(b) − F(l) = F(b) − F(a) It shows that only limits a and b are useful. Rest intermediate points are cancelled by each other. Consider a function f(x) = x that is undergoing integration (bar summation) in [−1, 1]. Take the width dx = 0.25, and corresponding integral steps n = 8. The bar diagram for integral of f(x) is given below:
  • 4. 4 Definite Integration x f b −1.00 b −0.75 b −0.50 b −0.25 b 0 b 0.25 b 0.50 b 0.75 b 1.00 The integral in summation form is given by I = 8 X i=1 f(xi) × dxi After solving the integral, we have I = 1 unit. Now, from the direct method of integral, if F is integral of function f then plot of F versus x shall be looked like as given below: x F b −1.00 b −0.75 b −0.50 b −0.25 b 0 b 0.25 b 0.50 b 0.75 b 1.00 b F(−1) = 0.5 b F(1) = 0.5 As we already discussed that, the definite integral depends only on the lower limit and upper limit of the integral. So, the I shall be equal to the sum of F(−1) and F(1). From the plot, I = 0.5 + 0.5 = 1. In case of multi-term function, i.e. I = 1 Z 0 (2x − 2) dx Never consider f(x) = 2x − 2 as a line whose slope is 2 and intercepts y-axis at −2 to avoid numerical errors. But consider it as I = 1 Z 0 2x dx − 1 Z 0 2 dx i.e. difference of areas covered by 2x and x-axis, and 2 and x-axis. Here, two areas of subtracted rather than finding the area of line curve 2x − 2
  • 5. 1.1. DEFINITE INTEGRATION 5 and x-axis. In later case, integral may be zero or negative depending on the values of two areas. Solved Problem 1.1 In the following plot, integral of function f, i.e. F and x graph is drawn. Find 1 Z 0 f(x) dx Again explain, why this value is negative. Solution 1 2 −1 1 2 3 −1 −2 x F b b b b b 3.0 0.0 -1.0 0.0 3.0 From the integral F = I = 1 Z 0 f(x) dx So, from definite integral 1 Z 0 f(x) dx = F(1) − F(0) From the plot, F(1) − F(0) = −1 − 0 = −1 Here, negative sign represents that there may be more than one terms in function f(x) whose integral F(x) is plotted here. During the integration of a multi-term function, result may be positive, zero or negative, depending on the values of area covered by first term of the function f(x) and area covered by second term of the function f(x).
  • 6. 6 Definite Integration 1.2 Properties A definite integral obeys some rules/properties which reduces integrals into easy to solve within the domain of limits. 1.2.1 In-dependent of Variable If function f(x) is independent of its independent variable, i.e. x, then b Z a f(x) dx = b Z a f(t) dt Proof R f(x) dx represents that we have to find the sum (integrate) of product of function f(x) and dx within the limits of x from a to b. Similarly, the relation R f(t) dt has same meaning except that, here independent vari- able is t. Both statements are true and projecting similar meaning. Hence b Z a f(x) dx = b Z a f(t) dt (1.2) Solved Problem 1.2 Transform the function in new variable s. The function is Z x sin x dx Solution Substitute x = s. It gives dx = ds. It converts the integral as Z x sin x dx = Z s sin s ds Solved Problem 1.3 Transform the function in new variable s. The function is Z sin x x dx
  • 7. 1.2. PROPERTIES 7 Solution Substitute x = s. It gives dx = ds. It converts the integral as Z sin x x dx = Z sin s s ds 1.2.2 Flipping of Limits If a and b > a are lower limit and upper limit respectively for a function f(x), where it is being integrated then function integration takes place from a to b. If the direction of integration is change then the integration of function becomes negative, i.e. b Z a f(x) dx = − a Z b f(t) dt Proof Definite integration of b R a f(x) dx is b Z a f(x) dx = F(b) − F(a) The right hand side of above relation shows that the value of integrated func- tion F(x) at lower limit a, i.e. F(a) is subtracted in the value of integrated function F(x) at upper limit b, i.e. F(b). Now take common in right hand side of above relation b Z a f(x) dx = −[−F(b) + F(a)] and it shows that from above description stated, upper limit is a and lower limit is b. Hence b Z a f(x) dx = − a Z b f(x) dx (1.3)
  • 8. 8 Definite Integration 1.2.3 Splitting of Limits Take, a and b as lower and upper limits for the integral of a function f(x). If F(x) is integral of f(x) as Z f(x) dx = F(x) and function f(x) is continuous through the region between [a, b]. We can add a third limit point between boundary limits, a → b, such as a → c → b, where a < c < b. In this case, integral of the function becomes b Z a f(x) dx = c Z a f(x) dx + b Z c f(x) dx (1.4) Proof Definite integration of b R a f(x) dx is b Z a f(x) dx = F(b) − F(a) Adding and subtracting function value, F(c), at a point c in above relation at right side. b Z a f(x) dx = F(b) − F(a) + F(c) − F(c) 1 1 2 x y a b 1 1 2 x y a b c Figure 1.1: This function is integrated in the closed limit of [a, b]. It rep- resents area of region enclosed between function and x-axis. Function is partitioned by splitting limit [a, b] in [a, c] and [c, b]. Area of the region in limit [a, b] is sum of area of these two regions.
  • 9. 1.2. PROPERTIES 9 Rearranging right hand side of above relation b Z a f(x) dx = [F(b) − F(c)] + [F(c) − F(a)] Rewriting right side in integral form: b Z a f(x) dx = b Z c f(x) dx + c Z a f(x) dx b Z a f(x) dx = c Z a f(x) dx + b Z c f(x) dx (1.5) Solved Problem 1.4 Find definite integral of each function as plotted in following figure. Solution 1 1 2 t y 1 1 2 3 t y In the first figure, curve has two line segments, (i) the line segment be- tween x = 0 to x = 1 and (ii) line segment between x = 1 to x = 2. The equation of lines are y1 = x; y2 = −x + 2 Here, slope of first line segment is +1 and second line segment is −1 and the y-axis intercepts are c = 0 and c = 2. Now the definite integral of the given curve is I = Z 2 0 y dx = Z 1 0 y1 dx + Z 2 1 y2 dx Substituting the values, we have I = Z 1 0 x dx + Z 2 1 (−x + 2) dx
  • 10. 10 Definite Integration Now, solving it, we get the definite result as I = 1 2 + 1 2 = 1 This is first result. In the second figure, the equations of line segments can be found by using principle of equation of line passes from two coordinate points. Thus the line segments between definite limits x = 0 to x = 1, x = 1 to x = 2 and x = 2 to x = 3 are respectively y1 = x; y2 = 1; y3 = −x 2 + 2 The integral is I = Z 1 0 y1 dx + Z 2 1 y2 dx + Z 3 2 y3 dx Substituting the values, we have I = Z 1 0 x dx + Z 2 1 1 dx + Z 3 2 (−x/2 + 2) dx Now, solving it, we get the definite result as I = 1 2 + 1 + 3 4 = 2.25 This is second result. 1 1 2 t y 1 1 2 3 t y These result gives the area bounded between curve and the x-axis. Solved Problem 1.5 Find definite integral of each function as plotted in following figure. Solution
  • 11. 1.2. PROPERTIES 11 1 1 2 t y 1 1 2 3 t y In the first figure, curve has two line segments, (i) the line segment be- tween x = 0 to x = 1 and (ii) line segment between x = 1 to x = 2. The equation of lines are y1 = −x + 1; y2 = x − 1 Here, slope of first line segment is −1 and second line segment is +1 and the y-axis intercepts are c = 1 and c = −1. Now the definite integral of the given curve is I = Z 2 0 y dx = Z 1 0 y1 dx + Z 2 1 y2 dx Substituting the values, we have I = Z 1 0 (−x + 1) dx + Z 2 1 (x − 1) dx Now, solving it, we get the definite result as I = 1 2 + 1 2 = 1 This is first result. In the second figure, the equations of line segments can be found by using principle of equation of line passes from two coordinate points. Thus the line segments between definite limits x = 0 to x = 1, x = 1 to x = 2 and x = 2 to x = 3 are respectively y1 = −0.5x + 1 y2 = 0.5x; y3 = −0.5x + 2 The integral is I = Z 1 0 y1 dx + Z 2 1 y2 dx + Z 3 2 y3 dx Substituting the values, we have I = Z 1 0 (−0.5x + 1) dx + Z 2 1 0.5x dx + Z 3 2 (−0.5x + 2) dx
  • 12. 12 Definite Integration Now, solving it, we get the definite result as I = 3 4 + 3 4 + 3 4 = 9 4 = 2.25 This is second result. 1 1 2 t y 1 1 2 3 t y These result gives the area bounded between curve and the x-axis. 1.2.4 Shifting of Variable If f(x) is a continuous function in [a, b] and limit exists everywhere in [a, b] then definite integration of the function is given by I = b Z a f(x) dx Now shift the function by sum of limits, i.e. x = a + b − t; dx = −dt The corresponding limits becomes, when x = a, t = b and when x = b, t = a. So, I = a Z b f(a + b − t) × −dt From the property of flipping of limits, we have I = b Z a f(a + b − t) dt
  • 13. 1.2. PROPERTIES 13 From the property of changing of independent variable t → x, we have I = b Z a f(a + b − x) dx Here, both relation represents to the same integral and have same meaning. This property is useful in finding of integral of relation type R = f(x) f(x) + f(k − x) Solved Problem 1.6 Evaluate 2 R 1 f(x) f(x)+f(3−x) dx. Solution Let I = 2 Z 1 f(x) f(x) + f(3 − x) dx (1.6) Put x = 1 + 2 − t, it gives dx = −dt. When x = 1, t = 2 and when x = 2, t = 1. Now, the integral relation becomes I = 1 Z 2 f(3 − t) f(3 − t) + f(t) × −dt Or I = 2 Z 1 f(3 − t) f(3 − t) + f(t) dt Transform t → x, we have I = 2 Z 1 f(3 − x) f(3 − x) + f(x) dt (1.7) Adding equations 1.6 and 1.7, we have 2I = 2 Z 1 f(3 − x) + f(x) f(3 − x) + f(x) dx
  • 14. 14 Definite Integration Or 2I = 1 ⇒ I = 1 2 This is desired result. Note that the answer of these types of relations is always half of the difference of upper and lower limits. 1 2 3 x y f(x) = sin x sin x+sin(3.14−x) 1 −1 x y f(x) = cos x cos x+cos(−x) The function f(x) should neither change its sign (from positive y-axis to negative y-axis or vice-versa) or nature (from sine to cosine or vice-versa) within the limits of integration. For example, this rule is applicable for f(x) = sin x sin x + sin(3.14 − x) but it is not applicable for f(x) = sin x sin x + sin(4 − x) as sin(4 − x) is negative till 4 − x > π. 1.2.5 Shifting of Periodic Function If f(x) is a periodic function in a range from 0 to a then the function f(x) can be written as a Z 0 f(x) dx = a Z 0 f(a − x) dx Proof sin(x) is a periodic function of 2π. After each interval of 2π, its phase is same to the phase, that was just before 2π value. Take a periodic function f(x) of period a. So, x = a − t; dx = −dt The corresponding limits becomes, t = a when x = 0 and t = 0 when x = a. Put the value of x, dx and corresponding limits in left side of relation
  • 15. 1.2. PROPERTIES 15 x y x f(x) x y x f(x − a) a Figure 1.2: f(x) is periodic function of period a, hence function height at x and at x ± a are same. Function f(x) and f(x ± a) have same meaning. a Z 0 f(x) dx = 0 Z a f(a − t) × (−dt) = − 0 Z a f(a − t) dt = a Z 0 f(a − t) dt Changing the variable t into x a Z 0 f(x) dx = a Z 0 f(a − x) dx (1.8) Split of Limit in Periodic Functions Take a sine function as shown below. This function is being integrate within limits of θ = 0 to θ = 2π. x y x f(x) b 0 b π x y x −f(x) b π b 2π Sign of periodic function changes from positive to negative and vice-versa continuously. For the period of θ = 0 to θ = π, sine function is in positive
  • 16. 16 Definite Integration region. While for the period of θ = π to θ = 2π, sine function is in negative region. Therefore to integrate the sine function in full range of θ = 0 to θ = 2π is zero. It shown that area covered between horizontal line and function is zero, which is impossible from the above figure. So, to get the actual result, we split the limits according to the nature of the function. So, 2π Z 0 f(θ) dθ = π Z 0 |f(θ)| dθ + 2π Z π | − f(θ)| dθ 1.2.6 Even & Odd Function x f(x) −a a sin x x f(x) −a a cos x Take the cases of sine and cosine as shown in above figure. f(x) = sin x is an odd function as we approach to negative side of x-axis, function approaches to negative y-axis within limits of −a ≤ x ≤ a. Therefore, it is not symmetrical about y-axis. Again, f(x) = cos x is an even function as we approach to negative side of x-axis, function function remains in positive y-axis within limits of −a ≤ x ≤ a. Therefore, it is symmetrical about y-axis, hence it is even function. A function is said to be even or odd function, if f(x) = ( Even if f(−x) = f(x) Odd if f(−x) = −f(x) If a function does not met with any of the above two conditions, then function is neither an odd function nor an even function. Now, take a function f(x), which is either an even function or an odd function within symmetrical limits from −a to +a. f(x) can be transform as a Z −a f(x) dx =      2 a R 0 f(x) dx if f(x) is even function 0 if f(x) is odd function (1.9)
  • 17. 1.2. PROPERTIES 17 Proof From left side of above definition a Z −a f(x) dx = 0 Z −a f(x) dx + a Z 0 f(x) dx (1.10) Now put x = −t in the first part of right hand side of the above relation then x = −t; dx = −dt The corresponding limits are obtained when x = −a then t = a. And when x = 0 then t = 0. The corresponding result is 0 Z −a f(x) dx = 0 Z a f(−t)(−dt) (1.11) = − 0 Z a f(−t) dt (1.12) = a Z 0 f(−t) dt (1.13) Case I : Even Function If f(x) is even function then f(−t) = f(t) the relation (1.10) becomes a Z −a f(x) dx = a Z 0 f(t)(dt) + a Z 0 f(x) dx And replacing t by x result is a Z −a f(x) dx = a Z 0 f(x)(dx) + a Z 0 f(x) dx x f(x) I1 I2 −a a cos x
  • 18. 18 Definite Integration From above figure, I = I1 + I2 where I represents to integral equivalent to area covered within the function and x-axis. Now, I1 = I2, therefore, I = 2I1 = 2I2. Therefore b Z a f(x) dx = 2 a Z 0 f(x) dx (1.14) Case II : Odd Function If f(x) is odd function then f(−t) = −f(t) the relation (1.10) becomes a Z −a f(x) dx = − a Z 0 f(t)(dt) + a Z 0 f(x) dx And replacing t by x result is a Z −a f(x) dx = − a Z 0 f(x)(dx) + a Z 0 f(x) dx x f(x) I1 I2 −a a sin x From above figure, I = −I1 + I2 where I represents to integral equivalent to area covered within the function and x-axis. Algebraic sum of these two areas is zero. Therefore I = 0, i.e. b Z a f(x) dx = 0 (1.15)
  • 19. 1.2. PROPERTIES 19 Integration of Even & Odd Functions Simply, take a function f(x) = xn which is defined as f(x) = ( Even if n is even Odd if n is odd On, its integration, we get I = Z f(x) dx = Z xn dx = xn+1 n + 1 Now, f(x) = ( Even if n is odd Odd if n is even It means, an odd function converts into an even function and vice-versa. Again, take the case of sin x, its integration is cos x. Here, sin x is an odd function which is transform into even function after integration. x f(x) sin x R x f(x) cos x Solved Problem 1.7 Find whether f(x) = x2 is an even function or an odd function. Solution x f(x)
  • 20. 20 Definite Integration A function f(x) is even if f(−x) = f(x) and odd if f(−x) = −f(x). Now, substituting the x by −x in the given function: f(−x) = (−x)2 = x2 = f(x) Therefore, the given function is an even function. Solved Problem 1.8 Find whether f(x) = −x2 is an even function or an odd function. Solution x f(x) A function f(x) is even if f(−x) = f(x) and odd if f(−x) = −f(x). Now, substituting the x by −x in the given function: f(−x) = −(−x)2 = −x2 = f(x) Therefore, the given function is an even function. Solved Problem 1.9 Find whether f(t) = t3 − 2t − 4 is an even function or an odd function. Solution A function f(t) is even if f(−t) = f(t) and odd if f(−t) = −f(t). Now, substituting the t by −t in the given function: f(−t) = (−t)3 − 2(−t) − 4 = −t3 + 2t − 4 = −(t3 − 2t + 4) 6= f(t) 6= −f(t) Therefore, the given function is neither an odd function nor an even function. Solved Problem 1.10 Find whether f(t) = sin t + t cos t is an odd function or an even function.. Solution A function f(t) is even if f(−t) = f(t) and odd if f(−t) = −f(t). Now, substituting the t by −t in the given function: f(−t) = sin(−t) + (−t) cos(−t) = − sin t − t cos t = −(sin t + t cos t) = −f(t) Therefore, the given function is an odd function.
  • 21. 1.2. PROPERTIES 21 Solved Problem 1.11 Show that integration of even function f(x) = x2 − 2 is results into odd function. Solution The given function f(x) = x2 − 2 is an even function. Now, integrating it, we have F(x) = Z f(x) dx = Z (x2 − 2) dx = x3 3 − 2x Now, the function is F(x) = x3 3 − 2x. To check the function is odd or even, substitute x by −x. F(x) = (−x)3 3 − 2(−x) = − x3 3 + 2x = − x3 3 − 2x = −F(x) Therefore, F(x) is an odd function. Solved Problem 1.12 Show that derivative of an odd function is an even function. Solution A function is said to be odd function if f(−x) = −f(x). Take simple odd function as f(x) = axn ⇒ f(−x) = −axn Where n is a positive odd integer, i.e. 1, 3, . . .. Using derivative rules, we have f ′ (x) = an xn−1 As n is a positive odd number, hence a × n will be positive. We know that if n is an odd number then n ± 1 is always even number. Now, n − 1 is definitely an even number. Now, f ′ (−x) = an(−x)n−1 = an xn−1 Hence, the derivative of an odd function is an even function. Solved Problem 1.13 Show that derivative of an even function is an odd function. Solution A function is said to be even function if f(−x) = f(x). Take simple even function as f(x) = axn ⇒ f(−x) = axn
  • 22. 22 Definite Integration Where n is a positive odd integer, i.e. 0, 2, 4, . . .. Using derivative rules, we have f ′ (x) = an xn−1 As n is a positive even number, hence a × n will be positive. We know that if n is an even number then n ± 1 is always odd number. Now, n − 1 is definitely an odd number. Now, f ′ (−x) = an(−x)n−1 = −an xn−1 Hence, the derivative of an even function is an odd function. Solved Problem 1.14 Show that integration of function f(x) = sin x is an even function. Solution The given function f(x) = sin x is an odd function. Now, integrating it, we have F(x) = Z f(x) dx = Z sin x dx = cos x Now, the function is F(x) = cos x. To check the function is odd or even, substitute x by −x. F(x) = cos(−x) = cos x = F(x) Therefore, F(x) is an even function. Solved Problem 1.15 Show that definite integral of odd function f(x) = sin x is zero in the domain of −π/2 ≤ x ≤ π/2. Solution The given function f(x) = sin x is an odd function. Now, integrating it, we have F(x) = Z f(x) dx = Z sin x dx = cos x Now the definite integral is F(x)| π/2 −π/2 = cos x| π/2 −π/2 = 0 Hence definite integral of odd function f(x) = sin x is zero in the given domain.
  • 23. 1.2. PROPERTIES 23 Solved Problem 1.16 Show that definite integral of even function f(x) = cos x in the domain of −π/2 ≤ x ≤ π/2 is twice to the definite integral of the function within the domain of −π/2 ≤ x ≤ 0 or 0 ≤ x ≤ π/2. Solution The definite integral of the given function in the domain of −π/2 ≤ x ≤ π/2 is I = π/2 Z −π/2 cos x dx = sin x| π/2 −π/2 = 2 Now, the definite integral of the same function in the domain of −π/2 ≤ x ≤ 0 or 0 ≤ x ≤ π/2 is I1 = 0 Z −π/2 cos x dx = sin x|0 −π/2 = 1 and I2 = π/2 Z 0 cos x dx = sin x|π/2 0 = 1 This shows that I is twice to the either I1 or I2. 1.2.7 Periodic Even Odd Function If f(x) is either an even periodic function or an odd periodic function of range from 0 to 2a then 2a Z 0 f(x) dx =      2 a R 0 f(x) dx if f(2a − x) = f(x) 0 if f(2a − x) = −f(x) (1.16) Proof From left side of above definition 2a Z 0 f(x) dx = a Z 0 f(x) dx + 2a Z a f(x) dx (1.17)
  • 24. 24 Definite Integration Now put x = (2a − t) and dx = −dt in the second term of the right hand side of the above relation alongwith corresponding limits are; when x = a, t = a and when x = 2a, t = 0. Now, the corresponding relation becomes 2a Z 0 f(x) dx = 0 Z a f(2a − t) (−dt) (1.18) = − 0 Z a f(2a − t) dt (1.19) = a Z 0 f(2a − t) dt (1.20) I If f(2a − x) is even periodic function then f(2a − t) = f(t) the relation (1.17) becomes 2a Z 0 f(x) dx = a Z 0 f(t) dt + a Z 0 f(x) dx And replacing t by x result is 2a Z 0 f(x) dx = a Z 0 f(x) dx + a Z 0 f(x) dx 2a Z 0 f(x) dx = 2 a Z 0 f(x) dx (1.21) II If f(2a − x) is odd function then f(2a − t) = −f(t) the relation (1.17) becomes 2a Z 0 f(x) dx = − a Z 0 f(t) dt + a Z 0 f(x) dx And replacing t by x result is 2a Z 0 f(x) dx = − a Z 0 f(x) dx + a Z 0 f(x) dx
  • 25. 1.2. PROPERTIES 25 2a Z 0 f(x) dx = 0 (1.22) 1.2.8 Splitting of Limit in n Parts If f(x) is a periodic function of period a then na Z 0 f(x) dx = n a Z 0 f(x) dx Proof The relation becomes na Z 0 f(x) dx = a Z 0 f(x) dx + 2a Z a f(x) dx + . . . . . . + na Z (n−1)a f(x) dx (1.23) Applying properties of the periodic function, when f(x) is even function, then the relation becomes na Z 0 f(x) dx = a Z 0 f(x) dx + a Z 0 f(x) dx + . . . . . . + a Z 0 f(x) dx (n times) na Z 0 f(x) dx = n a Z 0 f(x) dx (1.24) It proved. Solved Problem 1.17 Solve π/2 R 0 √ sin x √ sin x+ √ cos x dx = π 4 . Solution The integral is π/2 R 0 √ sin x √ sin x+ √ cos x dx which is a periodic function in the interval of θ = 0 to θ = π 2 . Now I = π/2 Z 0 √ sin x √ sin x + √ cos x dx (1.25)
  • 26. 26 Definite Integration And its periodic derivation is I = π/2 Z 0 p sin(π/2 − x) p sin(π/2 − x) + p cos(π/2 − x) dx I = π/2 Z 0 √ cos x √ cos x + √ sin x dx (1.26) Now adding above two equations 2I = π/2 Z 0 √ cos x + √ sin x √ cos x + √ sin x dx = π/2 Z 0 1dx Taking integration and substituting the limits of integration 2I = [x]π/2 0 = π/2 Now solving it we will get the result as I = π 4 (1.27) Solved Problem 1.18 Solve π/2 R 0 sin2 x sin x+cos x dx = 1 √ 2 loge( √ 2 + 1). Solution Solved Problem 1.19 Solve π R 0 x tan x sec x+tan x dx = π(π 2 − 1). Solution The integral is π R 0 x tan x sec x+tan x dx which is a periodic function in the interval of θ = 0 to θ = π. Now I = π Z 0 x tan x sec x + tan x dx (1.28)
  • 27. 1.2. PROPERTIES 27 And its periodic derivation is I = π Z 0 (π − x) tan(π − x) sec(π − x) + tan(π − x) dx I = π Z 0 (π − x) (− tan x) − sec x − tan x dx (1.29) Now adding above two equations 2I = π Z 0 π tan x sec x + tan x dx Rationalizing right hand side by multiplying and diving it by sec x − tan x. 2I = π Z 0 π tan x(sec x − tan x) sec2 x − tan2 x dx = π Z 0 π tan x(sec x − tan x) 1 dx Opening the bracket to make two parts in right hand side. 2I = π Z 0 π tan x · sec x dx − π Z 0 tan2 x dx = π Z 0 π tan x · sec x dx − π Z 0 (sec2 x − 1) dx Taking integration and substituting the limits of integration 2I = π [sec x]π 0 − π [tan x − x]π 0 = π [sec π − sec 0] − π [tan π − π − tan 0 + 0] = π [−1 − 1 − 0 + π + 0 + 0] = π [π − 2] Now solving it, we will get the result as I = π π 2 − 1 (1.30)
  • 28. 28 Definite Integration Solved Problem 1.20 Evaluate π R 0 ecos θ · cos(sin θ) dθ. Solution The given integral is I = π Z 0 ecos θ · cos(sin θ) dθ It is definite integral whose limits ranges from θ = 0 to θ = 2π. Now changing the term ecos θ · cos(sin θ) in exponential form. ecos θ · cos(sin θ) = Real part of {ecos θ · ei sin θ } Here cos(sin θ) is taken from relation ei sin θ = cos(sin θ) + i sin(sin θ) ei sin θ has both real and imaginary parts but according to question only real part is required in our problem. Now π Z 0 ecos θ · cos(sin θ) dθ = Real part of    π Z 0 ecos θ · ei sin θ dθ    = Real part of    π Z 0 ecos θ+i sin θ dθ    = Real part of    π Z 0 eexp(iθ) dθ    Substitute eiθ = z and eiθ i dθ = dz π Z 0 ecos θ · cos(sin θ) dθ = Real part of    θ=π Z θ=0 1 i ez z dz    = Real part of    θ=π Z θ=0 1 i 1 z 1 + z + z2 2! + . . . . . . dz    = Real part of ( 1 i log z + z + z2 2 × 2! + . . . . . . θ=π θ=0 )
  • 29. 1.2. PROPERTIES 29 Substituting the value of z π Z 0 ecos θ · cos(sin θ) dθ = Real part of 1 i iθ + eiθ z + e2iθ 2 × 2! + . . . . . . π 0 = Real part of θ + eiθ i + e2iθ 2 × 2! × i + . . . . . . π 0 Taking only real terms in right hand side π Z 0 ecos θ · cos(sin θ) dθ = θ + sin θ + sin 2θ 2 × 2! + . . . . . . π 0 = π It is the required answer. Addition of Constant Term in Integral Result In indefinite integration, after each integration we add a constant value for proper result. But the question is why do we do so. The answer is that, in indefinite integration we take reference point “zero” for a variable. Hence range of integration lies between zero to variable itself. If we want to change the reference point other than zero, we need two values. (i) new reference point, ‘c’ and (ii) function value, f(c) at this new reference point. The function value at this new reference point, ‘c’, can be found by just putting the new reference point value in function f(x). This constant value is work as new reference function. In definite integration the reference point other than zero is represented by lower limit and reference function value is represented by the lower limit integral value. Mathematically I = Z f(x) dx is infinite integral. Its lower limit is zero and upper limit is variable of integral itself. Hence in definite integral form, indefinite integral can be written as I = x Z 0 f(x) dx If we want to change the reference point other than zero, we have to add a constant term in right hand side. Let the reference point is c in x-line
  • 30. 30 Definite Integration where 0 c x then above integral becomes I = c Z 0 f(x) dx + x Z c f(x) dx here c R 0 f(x) dx is definite integral for x-limits ranging from zero to constant ‘c’. If it is a constant value then I = x Z c f(x) dx + C There is another reason that can interpret it. Suppose a function is like f(x) = g(x) + C After differentiation it gives F(x) = G(x) Where F(x) = d dx f(x) and G(x) = d dx g(x). If we again integrate above differentiated result then we get the actual function f(x) f(x) = Z F(x) dx = Z G(x) dx But after integration, the result will be f(x) = g(x) It differs from the actual function as there is no constant term. To make it actual function value, we add a constant term in right hand side. So f(x) = g(x) + C This is reason for out question. In case of definite integration, lower limit is act as reference point, hence there is no need to add a constant term in
  • 31. 1.2. PROPERTIES 31 right hand side of the answer. For example I = x Z c f(x) dx Solved Problem 1.21 If f(x) = x6 − x3 + 15, then find 1 R 0 f ′′ (x) dx. Solution The second derivative of the given function is f ′′ (x) = 30x4 − 6x Now, I = 1 Z 0 30x4 − 6x dx = 6x5 − 3x2 1 0 This gives I = 3. Solved Problem 1.22 If f(x) = tx3 − t2 x2 + 5, then find 1 R 0 f ′′ (x) dt. Solution The second derivative of the given function is f ′′ (x) = 6tx − 2t2 Now, I = 1 Z 0 6tx − 2t2 dt = 3xt2 − 2 3 t3 1 0 This gives I = 3x − 2 3 This is desired answer.
  • 32. 32 Definite Integration Solved Problem 1.23 A resonance term f(t) = sin(ωt) is derivated twice with respect to t and then integrated with respect to ω. Find the result. Solution The given function is f(t) = sin(ωt) Its twice derivative about t is given by f ′′ (t) = −ω2 sin(ωt) Now, integral is I = Z −ω2 sin(ωt) dω On integration it with respect to ω, we have I = − 2tω sin(ωt) + (2 − t2 w2 ) cos(ωt) t3 This is desired result. 1.2.9 Definite Integration Limits A definite integration of a function f(x) withing bounded region [a, b] is given by I = b Z a f(x) dx Here, a is lower bound of the function and b is upper bound of the function. If function f(x) is integrated within symmetric boundaries as I = a Z −a f(x) dx then it can be represented at with limits as I = lim ε→a ε Z −ε f(x) dx
  • 33. 1.2. PROPERTIES 33 This notation is helpful, when function is defined at a point but integration is being done in a range of limits. Integration of function at points 0 is written as I = lim ε→0 ε Z −ε f(x) dx and at point ∞ is written as I = lim ε→0 1/ε Z −1/ε f(x) dx Solved Problem 1.24 Evaluate limε→0 ε R −ε e−x dx. Solution This integral is symmetric integral near about zero. The inte- grand will represent a line of zero width and certain height. So, even if the height of the function is one unit at ε = 0, yet the area is zero due to zero width. Now, I = lim ε→0 ε Z −ε e−x dx = lim ε→0 h −e−x
  • 34.
  • 35. ε −ε i Or I = lim ε→0 eε − e−ε x f x f x f Figure 1.3: The symmetry of integration in above three figures is for |ε| → 0.5, |ε| → 0.25 and |ε| → 0.05. Expanding the exponential and simplifying them, we get I = lim ε→0 ε + −ε3 3! + . . . = 0
  • 36. 34 Definite Integration This is desired answer. 1.3 Average Value We know that R f(x) dx represents the Area between function f(x) and two points of width dx in base line. If b Z a f(x) dx is area between f(x) and x-limits (x = a and x = b). The average value for this limit is given by Iavg = b R a f(x) dx b R a dx (1.31) Above relation is similar to the mathematical average, i.e. average area is the ratio of total area to the difference of limit. Solved Problem 1.25 Find the average value of constant ‘c’ for the limits ranges from t = a to t = b. Solution The average value of function f(t) is given by Iavg = b R a c dt b R a dt t c b b c c b b a b Now Iavg = c [t]b a [t]b a
  • 37. 1.3. AVERAGE VALUE 35 Solving the relation Iavg = c This shows that the average value of constant remains constant. Solved Problem 1.26 Find the average value of f(t) = t2 for the t-limits ranges from t = a to t = b. Solution The average value of function f(t) is given by Iavg = b R a t dt b R a dt t f(t) b b f(a) f(b) b b a b t line Now Iavg = h t2 2 ib a [t]b a Solving the relation Iavg = 1 2 (a + b) This shows that the average values of a line is average of sum of two ends of the line. Solved Problem 1.27 Find the average value of f(x) = x2 for the x-limits ranges from x = 2 to x = 3. Solution From the average relation Iavg = b R a f(x) dx b R a dx
  • 38. 36 Definite Integration x f(x) b b f(a) f(b) b b a b f(x) = x2 Substituting the values and integrating for the limits Iavg = 3 R 2 x2 dx 3 R 2 dx Or Iavg = h x3 3 i3 2 [x]3 2 Or Iavg = 19 3 This is the required average value. Solved Problem 1.28 Find the average value of f(x) = sin(x) within the angle limits in radian values from x = π/3 to 2π/3. Solution The average of function within the limits is Iavg = 2π 3 R π 6 sin(x)dx 2π 3 R π 6 dx On integration and solving it Iavg = [− cos(x)] 2π 3 π 6 [x] 2π 3 π 6
  • 39. 1.4. PIECE-WISE FUNCTIONS 37 x f(x) b b f(π/6) f(2π/3) b b π/6 2π/3 f(x) = sin(x) Iavg = − cos(2π 3 ) + cos(π 6 ) 2π 3 − π 6 Or Iavg = 0.5 + 0.866) 2.093 − 0.523 = 1.366 1.57 = 0.87 This is required result. 1.4 Piece-wise Functions A function F(x) is said to be a piece-wise function if it follows different functions at different conditions. See the following function F(x), that is defined for two conditions of independent variable x, i.e. (i) when x 0 and (ii) x ≥ 0. F(x) = ( f(x) if x 0 g(x) if x ≥ 0 (1.32) The integration of the piece-wise functions is performed within the domain of the functions. Integration of above function F(x) is given by Z f(x) dx = 0 Z −∞ f(x) dx + ∞ Z 0 g(x) dx (1.33) Solved Problem 1.29 Find the integral of the given function. f(x) = ( 2 if − 4 ≤ x 0 x if 0 ≤ x ≤ 2
  • 40. 38 Definite Integration Solution Integration of given function is I = 2 Z −4 f(x) dx Replacing the function f(x) for its domain of ‘x’ I = 0 Z −4 2 dx + 2 Z 0 x dx On solving it I = [2x]0 −4 + x2 2 2 0 Or I = 10 Solved Problem 1.30 Find the integral of the given function in −∞ x ∞. f(x) = ( x − 1 if − 1 ≤ x ≤ 0 0 otherwise Solution 1 −1 −2 1 −1 −2 x f(x) 1 −1 −2 1 −1 −2 x F(x)
  • 41. 1.4. PIECE-WISE FUNCTIONS 39 The integral of the piecewise function is F(x) = ∞ Z −∞ f(x) dx Substituting the function in above integral relation for as it is defined piece- wise. F(x) = −1 Z −∞ 0 dx + 0 Z −1 (x − 1) dx + ∞ Z 0 0 dx On solving it F(x) = x2 2 − x 0 −1 = 0 − (−1)2 2 − (−1) = 3 2 It is integral of given piecewise function in domain −∞ x ∞. Solved Problem 1.31 Find the integral of the given unit step function within the domain of 0 ≤ t ≤ ∞. u(t − 2) = ( 0 if t 2 1 if t ≥ 2 What will be integration of e−at u(t − 2) within the same domain of ‘t’? Solution A unit step function is that function whose value is zero if variable is less than the transition point. And its value is ‘1’ if the variable is equal or larger than the transition point. In the given function, u(t − 2) tells that the transition point for the variable t is t − 2 = 0 ⇒ t = 2. Therefore, the unit step function value is 0 for t 2 and 1 for t ≥ 2. 1. Integration of given function is I = ∞ Z 0 u(t − 2) dt Replacing the unit step function u(t − 2) for its domain of ‘t’ I = 2 Z 0 0 dt + ∞ Z 2 1 dt
  • 42. 40 Definite Integration On solving it I = 0 + [x]∞ 2 Or I = ∞ Ans-1. 2. Integration of e−at u(t − 2) is I = ∞ Z 0 e−at u(t − 2) dt Replacing the unit step function u(t − 2) for its domain of ‘t’ I = 2 Z 0 e−at × 0 dt + ∞ Z 2 e−at × 1 dt On solving it I = 0 + e−at −a ∞ 2 Or I = e−2a a Ans-2. By this way, we can obtain the Laplace Transformations of unit step function also. Second derivative is very important in engineering. Solved Problem 1.32 Check whether the given piece-wise function exists or not. f(t) = ( 2t if 0 ≤ t ≤ 1 t + 1 if 1 ≤ t ≤ 2 Also find the ∞ R −∞ f(t) dt. Solution Function is critical for both piece-wise parts at t = 1. Therefore, at this point, function should be continuous for its existence. So f(t)− = 2 × 1 = 2
  • 43. 1.4. PIECE-WISE FUNCTIONS 41 and f(t)+ = 1 + 1 = 2 Here, f(t)+ = f(t)− , hence function exists. It is also necessary condition for the existence of the piece-wise function. 1 2 3 1 2 t f(t) (1) 1 2 3 1 2 t f(t) (2) 1 2 3 1 2 t f(t) (3) Figure 1.4: Figure (1) is plot of piece-wise function, while figure (2) is plot of integral of the given function. Figure (3) shows the total area covered by given piece-wise function and x-axis. Now, integration of the given function is I = ∞ Z −∞ f(t) dt The domain of the given function is not universal, therefore, the integral limits are depends on the sub-functions of piece-wise function. So, I = 1 Z 0 2t dt + 2 Z 1 (t + 1) dt Or I = [t2 ]1 0 + t2 2 + t 2 1 On solving it, we get I = 7/2. This is desired result.
  • 44. 42 Definite Integration Solved Problem 1.33 Find the integration of function as shown below: Solution 1 1 2 3 x y This piecewise function is a constant function as f(x) = 1 within the limits of integration 1 ≤ x ≤ 3. Now, its integration is I = 3 Z 1 1 dx = x|3 1 = 3 − 1 = 2 This is result. Solved Problem 1.34 Find the integration of function as shown below: Solution x y a a + k This piecewise function is a constant function as f(x) = c within the limits of integration a ≤ x ≤ a + k. Now, its integration is I = a+k Z a c dx = c h x|a+k a i = ck This is result.
  • 45. 1.4. PIECE-WISE FUNCTIONS 43 Solved Problem 1.35 Find the integration of function as shown below: Solution 1 −1 1 2 3 4 5 x y This piecewise function is a square wave function as |f(x)| = 1. f(x) = 1 within the limits of integration 1 ≤ x ≤ 2, f(x) = −1 within the limits of integration 2 ≤ x ≤ 3, f(x) = 1 within the limits of integration 3 ≤ x ≤ 4 and so on. Assuming that the integral is area integral, then I = 4 2 Z 1 1 dx = 4 x|2 1 = 4 This is result. Solved Problem 1.36 Sketch the graph of function u(t − 1) = 0 if t 1 1 if 1 ≤ t ≤ 3 and find the integral of it. Solution The sketch of the graph of above piecewise function is as given below: 1 1 2 3 t u This piecewise function is a unit step function which makes step at t = 1.
  • 46. 44 Definite Integration Now, its integration is I = 1 Z −∞ 0 dt + 3 Z 1 1 dt = t|3 1 = 3 − 1 = 2 This is result. Solved Problem 1.37 A ramp function is given by f(x) =    0 if x 0 mx if 0 ≤ x ≤ a 1 if a x ≤ a + k Find its integral in symbolic form. What shall be the integral value if (i) a = 2, k = 1 and (ii) a = 3, k = 0.25? Solution x f(x) a a + k b 1 α The integral of this ramp function shall be I = ∞ Z −∞ f(x) dx Substituting the piecewise function here, we have I = 0 Z −∞ 0 dx + a Z 0 mx dx + a+k Z a 1 dx On integrating it, we have I = c + m × x2 2
  • 47.
  • 48.
  • 49.
  • 51. 1.4. PIECE-WISE FUNCTIONS 45 As the function is zero for x 0, so c = 0. So, I = ma2 2 + k This is symbolic form of result of the given integral. Now 1: a = 2 and k = 1 I1 = m × 22 2 + 1 = 2m + 1 2: a = 3 and k = 0.25 I1 = m × 32 2 + 0.25 = 4.5m + 0.25 These are desire results. Solved Problem 1.38 A piecewise function is defined as f(x) = 1/k if a ≤ x ≤ a + k 0 otherwise Show that area covered by this function is unity, i.e. its integral is unity irrespective of value of k. Plot its graph. Solution The integral of this function is I = ∞ Z −∞ f(x) dx Substituting the piecewise function here, we have I = a Z −∞ 0 dx + a+k Z a 1 k dx + ∞ Z a+k 0 dx On integrating it, we have I = c + 1 k × x|a+k a + d
  • 52. 46 Definite Integration Here, c and d are arbitrary constants. As the function is zero for x a and x a + k, so c = 0 and d = 0. So, I = 1 k × k = 1 This result is independent of k, so integration of the given function is always unity irrespective of value of k. The plot of the given function is 1/k a a + k Figure 1.5: Plot of function f(x) = 1/k with k = 0.75. Solved Problem 1.39 A piecewise function is defined as f(x) = 1/100 if k ≤ x ≤ k + 100 0 otherwise Show that its integral is unity. Plot its graph. Solution The integral of this function is I = ∞ Z −∞ f(x) dx Substituting the piecewise function here, we have I = a Z −∞ 0 dx + k+100 Z k 1 100 dx + ∞ Z k+100 0 dx On integrating it, we have I = c + 1 100 × x|k+100 k + d
  • 53. 1.5. INTEGRAL IDENTITIES 47 Here, c and d are arbitrary constants. As the function is zero for x k and x k + 100, so c = 0 and d = 0. So, I = 1 100 × 100 = 1 This shows that result is unity. The plot of the given function is 1/100 k k + 100 Figure 1.6: Plot of function f(x) = 1/k with k = 100. 1.5 Integral Identities Let f(x) is a n degree function represented by In. The integration of this function within limits a to b is given by In = b Z a f(x) dx The right hand side of above relation can be written in terms of I and n only as In = αIj + βIk + γ Where Ij and Ik are the integral of the same function within the same range of limits as of In and having degrees other than the n. Solved Problem 1.40 Find the trigonometric identity for the integral In = π/2 Z 0 sinn x dx
  • 54. 48 Definite Integration Solution Let the given function is In = π/2 Z 0 sinn x dx Resolute right hand side for integral of sine function and obtaining the inte- gral similar to given in problem but have lesser degree. In = π/2 Z 0 sinn−2 x sin2 x dx Converting the sin2 x into 1 − cos2 x and In = π/2 Z 0 sinn−2 x 1 − cos2 x dx Or In = π/2 Z 0 sinn−2 x dx − π/2 Z 0 sinn−2 x cos2 x dx First term in right hand side of above relation is similar to the given integral except that it has degree of (n − 2), now In = In−2 − π/2 Z 0 sinn−2 x cos2 x dx Second term in right hand side of above integration is a beta function if n−2 is positive value. Now it gives In = In−2 − β 3 2 , n−1 2 2 It is the required identity of the given function.
  • 55. 1.5. INTEGRAL IDENTITIES 49 Solved Problem 1.41 Find the trigonometric identity for the integral In = π/2 Z π/4 cotn x dx Solution Let the given function is In = π/2 Z π/4 cotn x dx Resolute right hand side for integral of cotangent function and obtaining the integral similar to given in problem but have lesser degree. π/2 Z π/4 cotn−2 x cot2 x dx Converting the cot2 x into csc2 x − 1 and In = π/2 Z π/4 cotn−2 x csc2 x − 1 dx Or In = π/2 Z π/4 cotn−2 x csc2 x dx − π/2 Z π/4 cotn−2 x dx Second term in right hand side of above relation is similar to the given integral except that it has degree of (n − 2), now In = −In−2 + π/2 Z π/4 cotn−2 x csc2 x dx In second term, substitute cot x = t. It gives − csc2 x dx = dt. Corresponding lower and upper limits are 1 to 0. Now it gives In = −In−2 + 0 Z 1 tn−2 dt
  • 56. 50 Definite Integration Assuming that n − 2 is positive. Hence In = −In−2 − 1 n − 1 It is the required identity of the given function. This identity is true for all values of n where n ≥ 2. Now rearranging the degree of integral as n → n+1. In+1 = −In−1 − 1 n This is required answer. 1.6 Special Integral In mathematics, there are two important operators, (i) gamma operator and (ii) factorial. A gamma operator is represented by symbol Γ and factorial is represented by | or ! symbol. For example, factorial of integer 10 is written either as 10! or as |10. Similarly, gamma operator of a number 10 is given by Γ(10). Mathematically |10 = 10 × 9 × . . . × 3 × 2 × 1 In a factorial expansion, last multiplier is one. Again, Γ(10) = 9 × 8 × Γ(8) Gamma operator and factorial has relation as n! = Γ(n + 1) Note that argument of factorial operator is always a positive integer while ar- gument of gamma operator is a real number. This is basic difference between factorial and gamma operator. In factorials, |10.25 and |−10 operations are invalid. 1.6.1 Definite Integral of Laplace Type Integration, of those functions which can be transformed into exponential form can be integrated and transform in form of Laplace Transformation. If
  • 57. 1.6. SPECIAL INTEGRAL 51 f(t) is a positive function of t and s is a constant variable, then integral of type Ilapace = L[f(t)] = ∞ Z 0 f(t)e−st dt (1.34) is known as Laplace integral and method is called Laplace Transformation. Transformation word is used as old variable t is transformed into new vari- able s, i.e. the function is transform from time domain (say t domain) into frequency domain (say s domain). Here e−st is Laplace operator, i.e. inte- gral kernel and f(t) is operand. Old and new variable may be denoted by other alphabetic letters and it does not matter in our solutions. In Laplace transform, eat is known as shift operator. If a is positive then s is shifted leftward, i.e. s + a. Similarly, if a is negative then s is shifted rightward, i.e. s + a. Solved Problem 1.42 Find the integral of e−at with respect to t within limits from t = 0 to t = ∞. Also find the integral value when a → ∞. Solution From the given problem I = ∞ Z 0 e−at dt = e−at −a
  • 58.
  • 59.
  • 60.
  • 61. ∞ 0 It gives I = 1 a Note that, here a is taken as positive value. If a is taken as negative value then function becomes eat which is divergent function. t f(t) a = 1 t f(t) a = 10 t f(t) a = 100 Figure 1.7: Plot of the graph of function f(t) = e−at for a = 1, a = 10 and a = 100 respectively. The plot drops rapidly just right to t = 0 as a increases to infinity.
  • 62. 52 Definite Integration The integral value at a → ∞ is given by Ia→∞ = 1 ∞ = 0 Solved Problem 1.43 Find the integral of t e−at with respect to t within limits from t = 0 to t = ∞. Solution From the given problem I = ∞ Z 0 te−at dt = − (at + 1)e−at a2 ∞ 0 It gives I = 1 a2 Note that, here a is taken as positive value. If a is taken as negative value then function becomes eat which is divergent function. t f(t) a = 1 t f(t) a = 10 t f(t) a = 100 Figure 1.8: Plot of the graph of function f(t) = t e−at for a = 1, a = 10 and a = 100 respectively. The plot become more flat rapidly as a increases to infinity. Solved Problem 1.44 Find the integral of e−at , te−at and t2 e−at about t within limits from t = 0 to t = ∞. Using the symmetry, find the integral of tn e−at . Solution According to the question, I1 = ∞ Z 0 e−at dt = e−at −a
  • 63.
  • 64.
  • 65.
  • 67. 1.6. SPECIAL INTEGRAL 53 I2 = ∞ Z 0 te−at dt = − (at + 1)e−at a2 ∞ 0 = 1 a2 I3 = ∞ Z 0 t2 e−at dt = − (a2 t2 + 2at + 2)e−at a3 ∞ 0 = 2 a3 From these three results, we find that the integral values are in order of In = n! an+1 Where, n is power of t and is an integer. So, the ∞ Z 0 tn e−at dt = n! an+1 This is desire result. Solved Problem 1.45 Let κ is an operator which means κ[f(x)] = t Z 0 f(x) dx Now, find the value of κ sin x and κ2 sin x. Solution The given relation is κ[f(x)] = t Z 0 f(x) dx Now, substituting the value of f(x) = sin x, we have κ[sin x] = t Z 0 sin x dx = [− cos x]t 0 It gives κ[sin x] = 1 − cos t. Now, κ2 [sin x] can be written as κ [κ[sin x]]. Therefore, f(x) becomes κ[sin x] and its value is 1 − cos x, (t is replaced by
  • 68. 54 Definite Integration x). Now κ2 [sin x] = t Z 0 (1 − cos x) dx = [x − sin x]t 0 It gives κ2 [sin x] = t − sin t. Solved Problem 1.46 Find the value of R ∞ 0 e−x2 dx. Solution To find the integral of this problem, substitute x2 = t. The corresponding 2x dx = dt. Now, when x = 0, t = 0 and when x = ∞, t = ∞. The integral becomes I = Z ∞ 0 t−1/2 e−t dt 2 Or I = 1 2 Z ∞ 0 t 1 2 −1 e−t dt = 1 2 × Γ 1 2 This gives I = √ π/2. 1.6.2 Definite Integral of Fourier Type We know that Laplace type integral of a function is given by IL = ∞ Z 0 f(x)e−sx dx in s-domain or time domain. If s is replaced iω or jω (where i2 = −1 or j2 = −1) then integral becomes Fourier type. IF = ∞ Z 0 f(x)e−iωx dx (1.35) Where ω = 2πf in frequency domain. Laplace type integral is related with Fourier type integral by s ←→ iω. For complete Fourier transform IF = ∞ Z −∞ f(x)e−iωx dx (1.36)
  • 69. 1.6. SPECIAL INTEGRAL 55 A discrete function can also be represented into continuous function by apply- ing Fourier Series method, i.e. converting Fourier type integral into discrete series. A function f(x) is equivalent to f(x) = a0 + ∞ X n=1 an cos(nx) + ∞ X n=1 bn sin(nx) (1.37) Where a0, an and bn are three Fourier constants which can be evaluated for definite limit range x = a to x = b by using following three relations. a0 = 1 b − a b Z a f(x) dx (1.38) an = 2 b − a b Z a f(x) cos(nx)dx (1.39) bn = 2 b − a b Z a f(x) sin(nx)dx (1.40) Solved Problem 1.47 Find the constant value of Fourier series integration of the given function f(x) = x within limits from 0 to π. Solution Constant term a0 in the Fourier series integration is given by a0 = 1 b − a b Z a f(x) dx Taking limits from 0 to π and function f(x) = x, the Fourier series constant integral term becomes to a0 = 1 π − 0 π Z 0 x dx Or a0 = 1 π x2 2 π 0 It gives a0 = π/2.
  • 70. 56 Definite Integration Solved Problem 1.48 Find the constant value of Fourier series integration of the given function f(x) = sin x within limits from 0 to π. Solution Constant term a0 in the Fourier series integration is given by a0 = 1 b − a b Z a f(x) dx Taking limits from 0 to π and function f(x) = sin x, the Fourier series con- stant integral term becomes to a0 = 1 π − 0 π Z 0 sin x dx Or a0 = 1 π [− cos x]π 0 It gives a0 = 2/π. Solved Problem 1.49 Find the constant value of Fourier series integration of the given function f(x) = x sin x within limits from 0 to π. Solution Constant term a0 in the Fourier series integration is given by a0 = 1 b − a b Z a f(x) dx Taking limits from 0 to π and function f(x) = x sin x, the Fourier series constant integral term becomes to a0 = 1 π − 0 π Z 0 x sin x dx Or a0 = 1 π [sin x − x cos x]π 0 It gives a0 = 1.
  • 71. 1.6. SPECIAL INTEGRAL 57 Solved Problem 1.50 Find the constant value of Fourier series integration of the given function f(x) = x2 within limits from 0 to π. Solution Constant term a0 in the Fourier series integration is given by a0 = 1 b − a b Z a f(x) dx Taking limits from 0 to π and function f(x) = x2 , the Fourier series constant integral term becomes to a0 = 1 π − 0 π Z 0 x2 dx Or a0 = 1 π x3 3 π 0 It gives a0 = π2 /3. 1.6.3 Specific Workout In the following section, we do and find the definite integral by applying conditions and will analysis our answers. Solved Problem 1.51 Find the definite integration of f(x) = sin x within limits from 0 to π/2. What would the answer if function is multiplied by x? Solution The definite integral as per given problem is I = π/2 Z 0 sin x dx = [− cos x]π/2 0 = [− cos(π/2)] − [− cos 0] This gives I = 1. Now, function is multiplied by x. So, new integral be I′ = π/2 Z 0 x × sin x dx = [x × − cos x]π/2 0 − π/2 Z 0 d dx x Z sin x dx dx
  • 72. 58 Definite Integration On simplification I′ = hπ 2 × − cos(π/2) i − [0 × − cos 0] − π/2 Z 0 [− cos x] dx Or I′ = [sin x]π/2 0 It gives, I′ = 1. Thus both results are same. Solved Problem 1.52 Find the definite integration of f(x) = cos x within limits from 0 to π/2. What would the answer if function is multiplied by x? Solution The definite integral as per given problem is I = π/2 Z 0 cos x dx = [sin x]π/2 0 = [sin(π/2)] − [sin 0] This gives I = 1. Now, function is multiplied by x. So, new integral be I′ = π/2 Z 0 x × cos x dx = [x × sin x]π/2 0 − π/2 Z 0 d dx x Z cos x dx dx On simplification I′ = hπ 2 × sin(π/2) i − [0 × sin 0] − π/2 Z 0 [sin x] dx Or I′ = π 2 − 0 − [− cos x]π/2 0 = π 2 + cos(π/2) − cos 0 It gives, I′ = π 2 − 1. Thus both results are not same. Solved Problem 1.53 Find the definite integration of f(x) = 1 1+x2 within limits from 0 to ∞. What would the answer if function is multiplied by x? Solution The definite integral as per given problem is I = ∞ Z 0 1 1 + x2 dx = tan−1 x ∞ 0 = tan−1 ∞ − tan−1 0
  • 73. 1.6. SPECIAL INTEGRAL 59 This gives I = π/2. Now, function is multiplied by x. So, new integral be I′ = ∞ Z 0 x 1 + x2 dx Put 1 + x2 = t. On differentiation, it gives, 2x dx = dt. Or it gives x dx = dt/2. The corresponding limits are t = 1 to t = ∞ respectively when x = 0 and x = ∞. On simplification I′ = 1 2 × ∞ Z 1 1 t dt Or I′ = 1 2 × [ln t]∞ 1 It gives that I′ is divergent in the given region of x. So, indefinite integral will be found by substituting the value of t and neglecting the limits of integration and adding a constant. I′ = ln(1 + x2 ) 2 + C Solved Problem 1.54 Find the definite integration of f(k) = e−sk within limits from 0 to ∞. Here s is a constant. What would the answer if function is multiplied by k? Solution The definite integral as per given problem is I = ∞ Z 0 e−sk dk = 1 −s × e−sk x ∞ 0 = 1 −s × e−s×∞ − e−s×0 If s 0 then e−s×∞ will be equal to zero. If s 0 say s = −ǫ then e−s×∞ will be equal to eǫ×∞ and it will be equal to ∞. Second case is not acceptable as integral in divergent, so taking first case (s 0) only, we have I = 1/s. Now, function is multiplied by k. So, new integral be I′ = ∞ Z 0 ke−sk dk
  • 74. 60 Definite Integration Or I′ = − (sk + 1) e−sk s2 ∞ 0 It gives I′ = 1/s2 . 1.6.4 Repetitive Integrals Some function are periodic function in which their original value can be found after two or more times integration. These types of functions can be solved by assuming integral as I. f(x) and g(x) are two functions which returns to itself on twice derivative or twice integral. Here, two functions, f(x) and g(x) are taken. Derivating twice to the functions f(x) and g(x), they gave d dx f(x) = f ′ (x); d dx f ′ (x) = f(x) d dx g(x) = g′ (x); d dx g′ (x) = g(x) Integrating twice to the same functions f(x) and g(x), we got Z f(x) dx = F(x); Z F(x) dx = f(x) Z g(x) dx = G(x); Z G(x) dx = g(x) Now, let I is integral solution of integral of product of these two functions, i.e. f(x) and g(x) about x. Now I = Z f(x) × g(x) dx (1.41) Now applying product rule of integration, we get I = f(x) Z g(x) dx ± Z d dx f(x) Z g(x) dx dx (1.42) On solving I = f(x) G(x) ± Z h f ′ (x) G(x) i dx (1.43)
  • 75. 1.6. SPECIAL INTEGRAL 61 Again integrating second part of above relation I = f(x) G(x)± f ′ (x) Z G(x) dx ± Z d dx f ′ (x) × Z G(x) dx dx (1.44) on solving I = f(x) G(x) ± f ′ (x) g(x) ± Z f(x) × g(x) dx (1.45) R f(x) × g(x) dx is equal to I, hence above equation can be written as I = f(x) G(x) ± f ′ (x) g(x) ± I (1.46) Here, sign of I in right hand side is determinant. It may cancel to I in left side or may added to it. Let the coefficient of I is k. So, kI = f(x) G(x) ± f ′ (x) g(x) (1.47) Here k is constant and integral solution I can be obtained by dividing right hand side by k. ± is used to describe method of finding integral step by step. Next is the solved problem that explains this method step by step. Solved Problem 1.55 Find the integral of ex sin(x). Solution Let I = Z ex sin(x) dx Using product rule, we have I = ex Z sin(x) dx − Z d dx ex × Z sin(x) dx dx On simplification, we have I = ex × − cos(x) − Z [ex × − cos(x)] dx Again solving the second part of right hand side: I = −ex cos(x) + ex Z cos(x) dx − Z d dx ex Z cos x dx dx
  • 76. 62 Definite Integration On simplification I = −ex cos(x) + ex sin(x) − Z ex sin x dx Or 2I = ex (sin x − cos x) Or I = ex 2 (sin x − cos x) This is required result. 1.6.5 Derivative of Integrals Let a continuous and differentiable function f(x) whose anti-derivative is F(x) and derivative of F(x) is f(x) within the definite limits from a to b. Here a b are the function of t. Now the integral is b Z a f(x) dx = F(a) − F(b) Now derivative of the given function with respect to t is d dt b Z a f(x) dx = d dt [F(b) − F(a)] = f(b) · d dt b − f(a) · d dt a Or d dt   b Z a f(x) dx   = f(b) · db dt − f(a) · da dt (1.48) This is the derivative of a function under the sign of definite integral. Solved Problem 1.56 Evaluate the relation d dx x2 Z 0 sin(t) dt
  • 77. 1.6. SPECIAL INTEGRAL 63 Solution Now from the relation d dt   b Z a f(x) dx   = f(b) · db dt − f(a) · da dt Here function f(t) = sin(t). The result of the above relation will be d dx x2 Z 0 sin(t) dt = f(x2 ) · d dx x2 − f(0) · d dt 0 = 2x sin(x2 ) − 0 = 2x sin(x2 ) It is required result. Solved Problem 1.57 Evaluate d dx x2 Z 0 p sin(t) + cos(t) dt Solution Now from the relation d dt   b Z a f(x) dx   = f(b) · db dt − f(a) · da dt Here function f(t) = p sin(t) + cos(t). The result of the above relation will be d dx x2 Z 0 p sin(t) + cos(t) dt = f(x2 ) · d dx x2 − f(0) · d dt 0 = 2x p sin(x2) + cos(x2) − 0 = 2x p sin(x2) + cos(x2) It is required result.
  • 78. 64 Definite Integration 1.6.6 Vector Integration In vector integration, functions are taken along with their direction. A vector ~ F(x) = x2 î + 2ĵ is in xy plane. Its integration about the variable x is given by I = Z ~ F(x) dx (1.49) The result has direction of the force. Few solved examples are given below for better understanding. Solved Problem 1.58 A force ~ F(x) = 3îN is acting on an object that is displaced in the direction of force. Now find the work done when object is displaced from x = 2m to x = 3m. Solution Work done by a force is a scalar quantity. Hence, there is a scalar product between force and displacement. At any instant of time, the displacement of the force is from x to x + dx. Now dW = ~ F · d~ x Force and displacement are in same direction. Work done during the dis- placement from x = 2m to x = 3m is Z dW = 3 Z 2 3î · dxî = 3 Z 2 3dx On solving it W = 3 [x]3 2 = 3 Work done by the force within displaced limits is 3J. Solved Problem 1.59 A wire of length 3 is carrying a current of ~ I = 2îA in the magnetic field ~ B = 3 × 10−3 ĵTesla. Find the magnetic force and its direction in the wire. Solution Current in a wire flows along its length, i.e. the direction of current is always same to the length of wire. Hence, orientation of wire and direction of current is in x-axis. Magnetic force on an element of the wire dl is given by d~ F = ~ I × ~ Bdl
  • 79. 1.6. SPECIAL INTEGRAL 65 Now substituting the values d~ F = 2î × 3 × 10−3 ĵdl Force on whole wire is Z d~ F = 3 Z 0 2 × 3 × 10−3 k̂dl Or ~ F = 6 × 10−3 3 Z 0 dl k̂ Now ~ F = 1.8 × 10−2 k̂N Force on wire is 1.8 × 10−2 N in the direction of positive z-axis. 1.6.7 Principal General Value Consider a definite integral I = b Z a f(x) dx Where a and b are lower and upper limits of the definite integration. Take another point c that lies between a and b in number line and gives limit infinity of function f(x) as lim x→c f(x) = ∞ Now, the Integral of function at this point, x = c, can not be obtained as function is discontinued here. To Find the integral of above function, integral should be re-written as lim σ→0   c−lσ Z a f(x) dx + b Z c+mσ f(x) dx  
  • 80. 66 Definite Integration In above relation we have split the limits from a to b into from a to c − lσ and from c + mσ to b. As at point x = c function goes to infinity hence we have just displace the limit point c to left hand side by c −lσ and right hand side c+mσ. Here l and m are arbitrary constants. The above result is called general value of the given integral when l 6= m. If l = m then the limit is called the principal value of the integral. And it is given by lim σ→0   c−lσ Z a f(x) dx + b Z c+lσ f(x) dx   1.6.8 Integral of Graphs A function when sketched within its boundary limits gives the detailed pic- ture of properties of itself. A graph may be linear, curve, step or unit func- tion. Before, we start the integration of a function sketched as graph, first we need to identify the function itself. For example, in figure given below, there are three functions, i.e. line, parabola and circle are plotted respectively from left to right. 1 1 2 x y 1 1 2 x y 1 1 x y Now to integrate the curve, we just need the equation that is represented by the graph. Limits of definite integration are lower and upper boundaries of the curve. Solved Problem 1.60 Integrate the function shown in the figure. 1 1 2 x y
  • 81. 1.6. SPECIAL INTEGRAL 67 Solution The graph is a straight line. Its equation si y = mx + c. Slope of the line is 0.5/2.5. Now y = 0.02x + c Again when x = 0, y = 0.5 and it gives c = 0.5. Now the equation of line is y = 0.02x + 0.5 Now integration of this line is I = 2.5 Z 0 y dx It gives, I = 2.5 Z 0 (0.02x + 0.5)dx I = 0.01x2 + 0.5x + k 2.5 0 Or I = 0.0625 + 1.25 = 1.3125 This is required integration. Solved Problem 1.61 Integrate the function shown in the figure. 1 1 2 3 x y Solution The graph is a group of two straight lines. Now, y1 = m1x + c1 and y2 = m2x+c2. Slope of the lines are m1 = 1/2 and m2 = −1. Equations of lines are y1 = 0.5x + c1 and y2 = −1x + c2
  • 82. 68 Definite Integration Using the initial value conditions, when x = 0, y1 = 0 and it gives c1 = 0. Similarly, when x = 3, y2 = 0 and it gives c2 = 3. Now y1 = 0.5x and y2 = −x + 3 The limits of integration are different for different lines. Hence I = 2 Z 0 y1 dx + 3 Z 2 y2 dx Substituting the values of y1 and y2 and integrating it I = 2 Z 0 0.5x dx + 3 Z 2 −x + 3 dx Second part is positive within the limits of integration, i.e. 2x 3, hence it does not need to take the absolute value of second part. I = 0.25 × 4 + − x2 2 + 3x 3 2 = 1 + − 9 2 + 9 + 4 2 − 6 It gives the value of I = 3/2. Solved Problem 1.62 Integrate the function shown in the figure. 1 1 2 3 x y Solution The graph is a group of two straight lines. Now, y1 = m1x + c1 and y2 = m2x + c2. Slope of the lines are m1 = 1/2 and m2 = 0. Equations of lines are y1 = 0.5x + c1
  • 83. 1.6. SPECIAL INTEGRAL 69 and y2 = c2 Using the initial value conditions, when x = 0, y1 = 0 and it gives c1 = 0. Similarly, when x = 2, y2 = 1 and it gives c2 = 1. Now y1 = 0.5x and y2 = 1 The limits of integration are different for different lines. Hence I = 2 Z 0 y1 dx + 3 Z 2 y2 dx Substituting the values of y1 and y2 and integrating it I = 2 Z 0 0.5x dx + 3 Z 2 1 dx Or I = 0.25 × 4 + [x]3 2 = 1 + [3 − 2] It gives the value of I = 2. Solved Problem 1.63 Integrate the function shown in the figure. Solution 1 −1 1 2 3 x y
  • 84. 70 Definite Integration The graph is a group of two straight lines. Now, y1 = m1x + c1 and y2 = m2x + c2. Slope of the lines are m1 = 0 and m2 = 0. Equations of lines are y1 = c1 and y2 = c2 Using the initial value conditions, when x = 0.5, y1 = 1 and it gives c1 = 1. Similarly, when x = 2, y2 = −1 and it gives c2 = −1. Now y1 = 1 and y2 = −1 The limits of integration are different for different lines. Hence I = 2 Z 0.5 y1 dx + 3 Z 2 y2 dx Substituting the values of y1 and y2 and integrating it I = 2 Z 0.5 1 dx +
  • 85.
  • 86.
  • 87.
  • 88.
  • 89.
  • 91.
  • 92.
  • 93.
  • 94.
  • 95.
  • 96. Linear integral gives are bounded between limits, function and baseline. Hence, negative area should be taken as its absolute value. I = [x]2 0.5 + | [−x]3 2 | = 2 − 0.5 + | − 3 + 2| It gives the value of I = 2.5. Solved Problem 1.64 Integrate the function shown in the figure. Solution 1 2 1 2 x y
  • 97. 1.6. SPECIAL INTEGRAL 71 The graph is equation of circle. And it is given by x2 + y2 = 4. Hence I = 2 Z 0 y dx Substituting the values of y integrating it I = 2 Z 0 √ 4 − x2 dx Using the direct integral method for above relation I = 22 2 sin−1 x 2 + x 2 s 1 − x2 22 #2 0 = 2 sin−1 2 2 + 2 2 s 1 − 22 22 − sin−1 0 2 − 0 2 s 1 − 02 22 # It gives the value of I = π. Solved Problem 1.65 A square function defined as piece-wise function. rect(x) = ( 1 when − a ≤ x ≤ a 0 otherwise Now, find +∞ R −∞ rect(x) dx. Solution The given integral is I = +∞ Z −∞ rect(x) dx Substituting the rect(x) function within given limits, we have I = −a Z −∞ 0 dx + a Z −a 1 dx + ∞ Z a 0 dx
  • 98. 72 Definite Integration Taking integral of it, we have I = [x]a −a = a − (−a) = 2a Hence the integral of rect(x) is 2a within the given limits. Solved Problem 1.66 A ramp function defined as piece-wise function. ramp(x) = ( 2x when 0 ≤ x ≤ 1 0 otherwise Now, find the integral of ramp(x) in domain (−∞, ∞). Solution The given integral is I = +∞ Z −∞ ramp(x) dx Substituting the ramp(x) function within given limits, we have I = 0 Z −∞ 0 dx + 1 Z 0 2x dx + ∞ Z 1 0 dx Taking integral of it, we have I = x2 1 0 = 1 − 0 = 1 Hence the integral of ramp(x) is 1 within the given limits. 1.6.9 Vertically symmetric Function A function f is said to be vertically symmetric if its graph may be divided into two equal halves by y-axis. Graph of a vertically symmetric function is shown below: 1 1 2 −1 −2 x y 1 −1 −2 x y + 1 1 2 x y
  • 99. 1.6. SPECIAL INTEGRAL 73 The y-axis divides area covered by vertical symmetric function f and x- axis into two equal parts. If area covered by the vertically symmetric function is A then its equal halves has area A/2. 1 1 2 −1 −2 x y A 1 −1 −2 x y A/2 + 1 1 2 x y A/2 The area region covered by the vertically symmetric function f and x- axis from x = −2 to x = −1 is equal to the area region covered by the vertically symmetric function f and x-axis from x = 1 to x = 2. Sometime this statement is also said that the area region covered by the vertically symmetric function f and x-axis in x −1 is equal to the area region covered by the vertically symmetric function f and x-axis in x 1. 1 −1 −2 x y = 1 1 2 x y Note that, if area covered between vertically symmetric function t(x) and x-axis in −∞ x ∞ is positive one unit, then function is called probability density function (pdf) as probability always lies in [0, 1]. Solved Problem 1.67 A vertically symmetric function covers area 2 above the x-axis in x = −∞ to x = ∞. If residual area in x −1.25 is 0.315, then find the area in x = 0 to x = 1.25. Solution We have vertically symmetric function whose area is 2. Now, it can be divided into two equal halves each having area 1.
  • 100. 74 Definite Integration 1 −1 −2 x y x −1.25 0.315 = 1 1 2 x y x 1.25 0.315 Due to symmetry, area in x −1.25 is equal to the area in x 1.25. 1 1 2 x y 1 − 0.315 0.315 So area in x = 0 to x = 1.25 is given by A = 1 − area(x 1.25) = 1 − 0.315 = 0.685 This is desired answer. Solved Problem 1.68 Find the area of covered by the function e−|x| in the region x −1. Solution 1 1 2 3 4 −1 −2 −3 −4 x y 2 − 0.3678 0.3678 The given function f(x) = e−|x| is a vertically symmetrical function about y-axis. Its domain is −∞ to ∞. Area of the region x −1 is equal to the A = total area − area(x −1) Now, total area covered by the function and x-axis is given by A′ = ∞ Z −∞ e−|x| dx
  • 101. 1.6. SPECIAL INTEGRAL 75 We shall break limits into two parts i.e. when (i) x is negative and (ii) x is positive so that absolute operator may be removed by replacing (i) x by −x when limit of x 0 and (ii) x by +x when limit of x ≥ 0 respectively. This is given below: A′ = 0 Z −∞ e−(−x) dx + ∞ Z 0 e−(+x) dx = 0 Z −∞ ex dx + ∞ Z 0 e−x dx On solving it, we have A′ = ex 1 0 −∞ + e−x −1 ∞ 0 = 2 Now area in desired region, i.e. x −1 is A = 2 − −1 Z −∞ e−|x| dx When we remove absolute operator, x will change to −x as limits of integra- tion are negative. So, A = 2 − −1 Z −∞ e−(−x) dx = 2 − −1 Z −∞ ex dx Or A = 2 − [ex ]−1 −∞ = 2 − 0.3678 = 1.6322 This is desired answer. Critical Region In vertically symmetric function who are asymptotic to x-axis, the maximum area is about x → 0, i.e. about y-axis. As x → ∞, region between two successive x decreases. In the following figure, function e−|x| is plotted in x ∈ [−∞, ∞]. We see that as |x| 2 area covered by function and x-axis is very less than the area covered by function and x-axis in |x| 1.
  • 102. 76 Definite Integration 1 1 2 3 4 −1 −2 −3 −4 x y 0.3678 0.3678 As |x| → ∞, area covered by function and x-axis tends to zero. 1 1 2 3 4 −1 −2 −3 −4 x y 0.1353 0.1353 1 1 2 3 4 −1 −2 −3 −4 x y 0.3678 2 − 0.3678 − 0.1353 0.1353 It shows that when x 2, area covered by function and x-axis in x ∈ [2, ∞] is 0.1353 which is very very less than total area covered by function and x-axis in x ∈ [−∞, ∞] which is 2 magnitude. Thus it may be excluded in our computation if we are computing definite region. So, this region which insignificant is called critical region. Critical region is also symmetric, i.e. it exists in both side of the curve and have equal magnitude. Solved Problem 1.69 In the plot of function e−|x| , significant area more than 5% of total area is to be calculated. Find the domain of x that must be included in computation. Solution The total area of function e−|x| and x-axis in x ∈ [−∞, ∞] is 2 unit. We have to calculate significant area more than 5% of total area. Thus we must exclude the area (A say) that is 5% of total area 2 unit. That is A = 2 × 5 100 = 0.1
  • 103. 1.7. EXPONENTIAL TRIGONOMETRIC FUNCTIONS 77 1 1 2 3 4 −1 −2 −3 −4 x y 0.05 2 − 0.1 0.05 Due to symmetry, it is distributed both side of curve. So that one side has insignificant area 0.1/2 = 0.05 units. We compute area for the given function in one side and find the limits of x. This limit shall be used for both side considering the symmetry. Now, 0.05 = ∞ Z x e−|x| dx = ∞ Z x e−x dx On solving it, we have − e−∞ − e−x = 0.05 ⇒ x = 2.9957 The function and x-axis include area 0.05 in one side in x ∈ [2.9957, ∞]. Due to symmetry the significant area is in x ∈ [−2.9957, 2.9957] as shown in above figure. 1.7 Exponential Trigonometric Functions Integration of those functions which include trigonometric and exponential operators are evaluated by first transforming trigonometric parts into expo- nential form, by using Euler’s equation eix = cos x + i sin x (1.50) It shall reduce to the given function into exponential form only. We can find the cosine and sine function in form of euler equation as cos x = eix + e−ix 2 (1.51) and sin x = eix − e−ix 2i (1.52)